TMC test bank wrong questions 6-10/NBRC

Ace your homework & exams now with Quizwiz!

Which of the following could result in an increase in pulmonary vascular resistance (PVR)? hyperoxia hypovolemia Correct Answer decreased cardiac output

EXPLANATIONS: (u) A. Hypoxemia, not hyperoxia, will result in an increase in PVR. (u) B. Hypovolemia will most likely result in a decrease in PVR. (c) C. Excessive PEEP can compress the pulmonary vessels and obstruct blood flow, resulting in an increase in PVR. (u) D. Decreased cardiac output results in decreased circulating volume and decreased PVR.

You are assisting a resident perform orotracheal intubation of an semiconscious adult patient in the emergency room. After three failed attempts by the resident to place the tube in the trachea, your efforts to ventilate the patient using a bagvalve- mask system fail. Which of the following would you recommend to the resident as the next course of action? A. carry out at least one more intubation attempt B. perform a percutaneous dilational tracheotomy C. suspend intubation efforts and continue bagging D. insert a laryngeal mask airway

In a "can't intubate, can't ventilate" scenario, it its best to follow the emergency pathway of the ASA difficult airway algorithm, which includes an initial attempt to restore ventilation via a laryngeal mask airway. The correct answer is: insert a laryngeal mask airway

Tactile fremitus would be reduced in all of the following conditions except: A. COPD B. pneumothorax C. pleural effusion D. pulmonary edema

Tactile fremitus would be reduces in COPD and a pneumothorax because the lung is overinflated. A pleural effusion would block and decrease the sounds coming from the lungs. The correct answer is: pulmonary edema

A 2-year old asthmatic child is placed on an inline small volume nebulizer while receiving volume control SIMV. Shortly after the treatment has started, an alarm on the servo-controlled humidifier is activated. Which of the following is the most likely cause for this alarm? A. empty water reservoir B. low gas temperature C. decreased humidity output D. clogged expiratory filter

The 4-6 L/min of additional unheated gas flow added to the circuit when using a small volume nebulizer will lower the temperature at the patient's airway, which can trigger a humidifier alarm. Either the humidifier's low temperature alarm should be readjusted during treatment or an electronic drug nebulizer (ultrasonic or mesh) should be used to avoid adding extra flow to the ventilator circuit. 455 The correct answer is: low gas temperature

While assessing a patient receiving artificial ventilatory support, you note wide swings in the central venous pressure (CVP) during mechanical breaths. Which of the following is the most likely cause of this finding? A. poor venous return B. pulmonary barotrauma C. decreased pleural pressure D. respiratory alkalosis

Wide swings in CVP during mechanical ventilation are most often caused by decreased venous return due to hypovolemia and worsened by positive pressure. The correct answer is: poor venous return

When calibrating a Clark electrode O2 analyzer, you cannot get the unit to read higher than 80% when the sensor is exposed to 100% O2. Your first action should be to: A. reset the alarms B. replace the batteries C. replace the membrane D. replace the electrode

When a Clark electrode O2 analyzer fails to accurately read 100% O2, the first step should be to check/replace the batteries. If fresh batteries do not solve the problem, likely the electrode needs replacement. The correct answer is: replace the batteries

A physician orders smoking cessation counseling for a 60-year-old male newly diagnosed with COPD. At the 1-month follow-up visit, the following ABG results are obtained: pH 7.38 PCO2 45 torr PO2 56 torr HCO3- 26 mEq/L Hb 16 g/dL O2Hb 82.1% COHb 5.5% MetHb 1.0% Regarding the success of the smoking cessation counseling, a respiratory therapist should conclude the results: reflect no smoking or environmental exposure. reflect smoking or environmental exposure. are inconclusive. reflect inaccurate data.

B. A COHb of >3% is indicative of continued smoking or environmental exposure.

When reviewing the chart of a newly admitted patient, a respiratory therapist finds the patient has COPD and a 70-pack-year smoking history. The patient was admitted for dyspnea. The patient is somnolent and can protect his airway. Which of the following should the therapist recommend FIRST? A. CT scan of the chest B. pulmonary function testing C. echocardiogram D. arterial blood gas analysis

(a) A. Although a CT scan of the chest may be indicated in the future, immediate intervention is required now. (u) B. The patient would be unable to perform pulmonary function testing due to the level of consciousness. (u) C. There is no evidence of primary heart disease, therefore an echocardiogram is not indicated at this time. (c) D. Arterial blood gas analysis is critical to guide the need for further intervention, such as NPPV and oxygen therapy.

A 65-year-old patient who is post-CABG was extubated 4 hours ago and complains of increasing shortness of breath. Breath sounds are decreased over the right lung field. The following data are available: HR 105 RR 25 BP 155/90 mm Hg SpO2 90% on 4-L nasal cannula Which of the following studies should a respiratory therapist recommend NEXT? A. bedside spirometry B. chest CT with contrast C. ventilation-perfusion scan D. chest radiograph

(a) A. While spirometry may be performed prior to surgery to determine the presence of pulmonary abnormalities, it will not provide useful diagnostic information this soon following extubation. (u) B. The patient's physiological data does not support an order for a chest CT as the next step. (u) C. The patient's physiological data does not support an order for a ventilation-perfusion scan as the next step. (c) D. The patient's complaints, in addition to the physiological data, suggests an abnormality that may be identified by a chest radiograph.

A patient with a history of chronic bronchitis complains of shortness of breath following thoracic surgery. The patient is receiving 30% O2 by aerosol mask. Rhonchi in the right lower lobe are heard on auscultation. Which of the following should a respiratory therapist recommend? A. vibratory PEP B. postural drainage C. dornase alfa (Pulmozyme) therapy D. manual percussion

(c) A. Deep breathing accompanied by PEP therapy will provide optimal bronchial hygiene to clear secretions. (u) B. Postural drainage by itself has little benefit. (u) C. Dornase alfa is not the first choice of therapy and is expensive. (h) D. Manual percussion is a relative contraindication in a patient following chest surgery because of chest tubes and incisional pain.

A 21-year-old man arrives in the ED after rescue from a house fire. Physical examination reveals burns on the upper chest and face, and marked edema of the face and oropharynx. Analysis results of an arterial blood gas sample obtained while the patient was breathing air are as follows: pH 7.55 PCO2 26 torr PO2 105 torr HCO3- 23 mEq/L BE +2 mEq/L Which of the following should a respiratory therapist recommend? A. Intubate the patient. B. Sedate the patient. C. Initiate levalbuterol (Xopenex) therapy. D. Administer cool aerosol therapy.

(c) A. Facial burns with oropharynx edema indicate upper airway burns and the potential for worsening edema and airway obstruction. Intubation is necessary to assure airway patency. (h) B. Although the arterial blood gas analysis indicates hyperventilation, the patient's airway should be protected before sedation is given. (u) C. Levalbuterol is a beta-agonist that relaxes bronchial smooth muscle. It will not decrease the laryngeal edema associated with facial burns. (u) D. Cool aerosol therapy will not reduce the edema and will further delay steps to protect the airway.

Which of the following is associated with an optimally exposed chest radiograph? A. Vertebrae and lung markings are clearly visible. B. Pleural spaces and heart borders are well defined. C. Pulmonary vascular markings are absent. D. Air bronchograms are easily identified

(c) A. In an optimally exposed chest radiograph, vertebrae are visible while maintaining visible lung markings. (u) B. The pleural space is not visible unless there is some pathology. (u) C. Pulmonary vascular markings should be visible. (a) D. Air bronchograms are not always present in a chest radiograph, and therefore should not be used to assess exposure.

While counseling a patient during a smoking cessation session, the patient expresses concern about weight gain. A respiratory therapist should address the patient's concern by explaining that this is partially the result of A. decreased metabolism. B. increased loss of self-control. C. reliance on nicotine replacement therapy. D. lack of available aversive conditioning

(c) A. Metabolism decreases when nicotine is withdrawn. If dietary intake remains unaltered and exercise is not included, there may be a resulting weight gain. (u) B. This message would be considered negative feedback. It will be more important to encourage someone who is trying to eliminate an addiction. (u) C. Nicotine replacement therapy will often reduce the withdrawal symptoms but should not be considered a reliable way to prevent weight gain. (u) D. It is not conclusive that any of these methods actually work. Usually incorporated into a smoking cessation program is positive reinforcement or a reward system.

Which of the following drugs is most appropriate to paralyze a sedated 30-year-old patient in status asthmaticus who is receiving continuous mechanical ventilation? A. rocuronium bromide (Zemuron) B. morphine C. midazolam (Versed) D. succinylcholine chloride (Anectine)

(c) A. Rocuronium bromide is a nondepolarizing skeletal muscle relaxant which may be administered to patients prior to endotracheal intubation, or during mechanical ventilation. (h) B. Morphine is an opioid which acts upon receptors in the brain and spinal cord to reduce the feeling of and emotional response to pain. (a) C. Midazolam is a nondepolarizing skeletal muscle relaxant whose adverse effects include hypersensitivity reactions associated with histamine release producing bronchospasm, flushing, erythema, acute urticaria, hypotension, and tachycardia. (u) D. Succinylcholine chloride is a depolarizing skeletal muscle relaxant whose onset of paralysis is very rapid (less than 1 minute after IV administration), and effects are very short (approximately 4 to 6 minutes). It is not an effective agent for skeletal muscle paralysis because it will not sustain paralysis.

A 24-year-old female presents with a history of nasal stuffiness, episodes of daytime dyspnea, and a cough that occurs every summer. Which of the following drug classifications should a respiratory therapist recommend to control the patient's symptoms? A. leukotriene inhibitor B. IgE immunoglobulin antagonist C. beta-adrenergic agonist D. anticholinergic

(c) A. The patient has allergic rhinitis and, at most, mild persistent asthma. Leukotriene inhibitors, such as montelukast (Singulair), are indicated to control mild asthma and allergic rhinitis. (h) B. IgE immunoglobulin antagonist, such as omalizumab (Xolair), is indicated for severe allergic asthma and carries significant risk. (u) C. Beta-adrenergic agonists are rescue drugs. They should not be used to control asthma and should have no role in the treatment of allergic rhinitis. (u) D. Short-acting antihistamines are part of the arsenal of treatment for allergic rhinitis; however, they have no affect on dyspnea. They also produce many systemic side effects.

A respiratory therapist is called to deliver a second dose of 5 mg aerosolized albuterol to a patient with acute asthma. The patient had audible wheezing after the first treatment 20 minutes ago. After the second treatment, auscultation reveals markedly diminished breath sounds. Which of the following should a respiratory therapist recommend? A. Begin continuous nebulized bronchodilator therapy at 10 mg every hour. B. Continue to administer 5 mg albuterol every 2 hours. C. Decrease albuterol dose to 2.5 mg every 2 hours. D. Change to ipratropium bromide (Atrovent) 0.5 mg every 6 hours.

(c) A. The patient is exhibiting refractory wheezing. The diminished breath sounds after the second treatment indicate worsening obstruction. Higher dose, continuous therapy may be more effective for this patient than intermittent dosing. (u) B. The decreased breath sounds and absent wheezing reflect the lack of air exchange. The current therapy is not effective and should be changed. Higher dose, continuous therapy may be more effective for this patient than intermittent dosing. (u) C. The patient's condition is worsening. More aggressive therapy is warranted. (u) D. While ipratropium bromide may be given in addition to albuterol, it would not be effective as solo therapy.

Which of the following can a respiratory therapist use to determine a patient's rapid-shallow breathing index? A. vane respirometer B. peak flowmeter C. pressure manometer D. capnometer

(c) A. The rapid-shallow breathing index is calculated using the formula: RR/(minute volume)(RR). A vane respirometer is used to determine the minute volume. (u) B. A peak flowmeter measures the peak expiratory flow, not minute volume. (u) C. A pressure manometer measures pressure, not minute volume. (u) D. A capnometer measures exhaled CO2, not minute volume.

While breathing spontaneously, a 54.5-kg (120-lb) female has a respiratory rate of 30 and a tidal volume of 200 mL. After a sedative was administered, her respiratory rate decreased to 10 and her tidal volume increased to 600 mL. Which of the following ventilatory parameters increased? A. minute ventilation B. alveolar ventilation C. mechanical dead space ventilation D. anatomic dead space ventilation

(c) B. The minute alveolar ventilation will increase (assuming a constant anatomic dead space of 1 mL/kg of body weight) because as the VT increases, the portion of each breath going to alveolar ventilation increases. (Note: V̇A = (VT - VD) x RR) Before sedation: V̇A = (200-120) mL x 30/min = 80 mL x 30/min = 2400 mL/min After sedation: V̇A = (600-120) mL x 10/min = 480 mL x 10/min = 4800 mL/min

Which of the following drugs is appropriate for the sedation of a 30-year-old patient with status asthmaticus and hypotension who is receiving mechanical ventilation? A. X cisatracurium (Nimbex) B. √ lorazepam (Ativan) C. vecuronium bromide (Norcuron) D. morphine

(h) A. Cisatracurium is a nondepolarizing skeletal muscle relaxant. (c) B. Lorazepam is a benzodiazepine used as a sedative to relieve anxiety. (h) C. Vecuronium bromide is a nondepolarizing skeletal muscle relaxant. (h) D. Morphine sulfate is an opioid whose adverse effects include hypotension.

A respiratory therapist determined a patient receiving mechanical ventilation developed auto-PEEP due to dynamic airflow obstruction. Which of the following should the therapist do? A. Add an inspiratory plateau. B. Add mechanical dead space. C. Decrease the mandatory rate. D. Decrease the inspiratory flow

(u) A. Breath-holding will not decrease the trapped air causing the auto-PEEP. (h) B. Adding mechanical dead space will not affect the time available for exhalation and may increase the arterial carbon dioxide. (c) C. Decreasing the mandatory rate will increase expiratory time. Allowing more time to exhale will reduce air trapped in the lungs at the end of exhalation, or the auto-PEEP. (u) D. Decreasing the inspiratory flow will decrease expiratory time, allowing less time for air to escape. This will not reduce auto-PEEP and has the potential to worsen air-trapping.

1. A 55-year-old patient with dyspnea and non-productive cough has the following examination results noted over the right lower lung field: - dullness to percussion - decreased tactile fremitus - diminished breath sounds Which of the following is most likely to produce these findings? A. pulmonary embolism B. pneumonia C. pneumothorax D. pleural effusion

(u) A. Decreased tactile fremitus and dullness to percussion are not associated with pulmonary embolism. (u) B. Pneumonia is associated with increased tactile fremitus, a productive cough, and bronchial breath sounds. (u) C. While pneumothorax is associated with decreased tactile fremitus and diminished breath sounds, it will not result in dullness to percussion. (c) D. These findings are consistent with a pleural effusion.

A wait of a few minutes should be expected after application of a transcutaneous electrode before values stabilize. This is associated with A. electrode calibration. B. capillary dilatation. C. barometric pressure equilibration. D. cerebral blood perfusion.

(u) A. Electrode calibration is performed prior to application of the electrode. (c) B. The site must be heated for 2-5 minutes to facilitate capillary dilatation. Once dilatation occurs, transcutaneous values will stabilize. (u) C. Barometric pressure is part of the calibration procedure performed prior to application to the skin. (u) D. Cerebral blood perfusion is not assessed during transcutaneous monitoring, and has no affect on the values.

A previously healthy 30-year-old patient is hospitalized with chills and fever. A chest radiograph is consistent with right upper lobe pneumonia. Which of the following is most likely to aid in the patient's management? A. insufflation-exsufflation therapy B. coached coughing and deep breathing C. bland aerosol therapy D. spirometry before and after a bronchodilator

(u) A. Insufflation-exsufflation therapy is used for patients with excessive secretions and weak cough effort, often seen in patients with neuromuscular diseases. (c) B. Coached coughing and deep breathing is the best method for mobilizing secretions in a young healthy adult. (a) C. Bland aerosol therapy may be indicated if secretions were evident at this stage of the patient's disease. (u) D. There is no evidence or history of bronchospasm in this case.

A gas is saturated with 100% relative humidity at 32.2º C (90º F). As the gas cools to 26.7º C (80º F), the relative humidity will A. increase by 10%. B. √ remain 100% and excess water will rain out. C. X decrease as a result of cooling of the gas. D. decrease as water condenses.

(u) A. It is impossible to attain relative humidity greater than 100%. (c) B. As the temperature drops and the gas cools, the relative humidity remains at 100%, causing water vapor to condense into liquid. (u) C. Relative humidity increases as a gas cools. When the relative humidity is at 100%, the gas is unable to hold more water vapor and it condenses into liquid ("rain out"). (u) D. Decreasing the temperature of a gas increases the relative humidity.

Initiation of an expiratory hold just prior to the next ventilator-delivered breath facilitates the measurement of A. Pplat. B. auto-PEEP. C. peak airway pressure. D. set PEEP.

(u) A. Pplat is measured during an inspiratory hold. (c) B. Auto-PEEP is measured by subtracting set PEEP from the measured pressure during an expiratory hold. (u) C. Peak airway pressure is measured when maximal inspiration is attained. (u) D. Set PEEP is the baseline pressure at exhalation.

A patient who is receiving mechanical ventilation is scheduled for a fiberoptic bronchoscopy. Which of the following is the primary threat to adequate ventilation during the procedure? A. secretion production B. elevated airway pressure C. loss of consciousness D. airway obstruction

(u) A. Secretions is an indication for fiberoptic bronchoscopy in the mechanically ventilated patient. (u) B. Elevated airway pressures are generally required to ventilate around the scope inside the airway. (u) C. Loss of consciousness will not directly impact adequate ventilation if compensatory measures are taken with ventilator settings. (c) D. Placing the scope in the artifical airway may obstruct the tube, especially if the scope is too large for the artificial airway.

A physician orders 1 mg of medication to be administered by aerosol. If the concentration of a stock solution is 0.04%, how many mL of the solution must be administered? A. 0.25 B. 0.40 C. 4.00 D. 2.50

(u) A. See D for correct calculation. (u) B. See D for correct calculation. (u) C. See D for correct calculation. (c) D. 0.04% = 0.4 mg/mL; divide the desired dosage (1 mg) by the actual concentration: 1 mg 0.4 mg/mL = 2.5 mL

A patient's blood pressure is monitored by an appropriately positioned arterial catheter transducer. The transducer is mounted on an IV pole that is not attached to the bed. The bed is lowered to permit routine patient care. How will this affect the measured blood pressure? systolic diastolic 1. increased increased 2. increased decreased 3. decreased decreased 4. decreased increased A. X 1 B. 2 C. √ 3 D. 4

(u) A. Systolic and diastolic pressures will increase if the bed is raised above the transducer. (u) B. Changes in the level of the bed relative to the transducer will make both systolic and diastolic pressures move the same direction. (c) C. A fluid-filled transducer catheter system is zeroed to the patient's position. Lowering the patient relative to the height of the transducer will result in both a lower systolic and diastolic blood pressure reading proportionate to the change in patient position.

Following placement of an 8.0-mm ID tracheostomy tube, how much air should be placed into the cuff? A. a minimum of 8 mL B. enough to produce a pressure of 35 cm H2O C. √ enough to prevent a leak at peak inspiration D. X enough to provide tension in the pilot balloon

(u) A. The appropriate volume of air in the cuff is variable and depends upon the amount required to attain a minimal occlusion pressure or a minimal leak. (u) B. The appropriate cuff pressure is variable and depends upon the amount of air required to attain a minimal occlusion pressure or a minimal leak. (c) C. Minimal occlusion pressure is an appropriate technique in which the cuff is inflated until no air is heard at peak inspiration. (u) D. Palpating the pilot balloon is not sufficient to detect appropriate cuff pressure.

A respiratory therapist is reviewing the medical record of an infant to determine whether any respiratory care is indicated in the home. Which of the following suggests the need for home apnea monitoring? A. albuterol aerosol every 4 hours prn B. caffeine citrate daily C. APGAR scores of 3 and 4 D. oxygen by cannula at 0.25 L/min

(u) A. The need for albuterol prn does not indicate the infant is at risk for apnea. (c) B. Caffeine citrate is used to treat apneic conditions and indicates the need for home apnea monitoring. (u) C. APGAR scores at birth do not have any correlation to the need for home apnea monitoring. (u) D. The need for home oxygen therapy is not directly related to the risk for apneic episodes.

A 20-year-old female with asthma presents in severe respiratory distress. The patient is tachypneic with accessory muscle use. Breath sounds reveal bilateral expiratory wheezes and SpO2 is 87% on air. A respiratory therapist should recommend initiating A. levalbuterol (Xopenox) by MDI. B. albuterol by continuous nebulization. C. salmeterol (Serevent) by dry powder inhaler. D. budesonide (Pulmicort) by small-volume nebulizer.

(u) A. The patient is in severe distress and will most likely be unable to coordinate or properly perform the inspiratory maneuver needed for MDI administration of a short-acting bronchodilator. (c) B. Continuous administration of albuterol will best address the severe respiratory distress the patient is experiencing associated with an acute exacerbation of asthma. Since the SpO2 is 87%, oxygen should be used to power the nebulizer. (h) C. Salmeterol is a long-acting bronchodilator and used for maintenance therapy. It is not an appropriate medication for use during an acute exacerbation. (u) D. Budesonide is an inhaled corticosteroid and will not provide quick relief from the bronchoconstriction and bronchospasm that are causing the respiratory distress.

78. While testing the proper function of a flow-inflating bag, a respiratory therapist notes the bag does not fully inflate with occlusion of the patient connector. Which of the following may be the cause of the problem? A. malfunctioning inlet valve B. excessive oxygen flow C. open flow control valve D. missing oxygen reservoir

(u) A. There is no inlet valve contained in a flow-inflating bag. (u) B. Excessive gas flow into the bag will not cause it to collapse. (c) C. The flow control valve in conjunction with the gas flow into the bag regulates the filling of the bag. With the flow control valve open, the gas going into the bag will immediately leave the bag through the open flow control valve. (u) D. There is no separate oxygen reservoir to the self-inflating bag.

If a respiratory therapist chooses to use a portable ventilator to transport a patient within a hospital, the ventilator should have the ability to A. monitor pulmonary compliance. B. sound a disconnect alarm. C. switch to CPAP mode. D. provide pressure-supported breaths.

(u) A. When transporting a patient on a portable ventilator, it is not necessary to monitor pulmonary compliance. (c) B. A disconnect alarm is vital to monitor when a patient is being transported using a portable ventilator. (u) C. CPAP is not likely to be used during patient transport within the hospital. (u) D. The ability for a portable ventilator to provide pressure-supported breaths is not necessary when transporting a patient within the hospital. Using other forms of ventilation such as VC ventilation can be used for such transport.

While doing a ventilator check on a patient receiving volume controlled ventilation you observe 'scalloping' of the inspiratory airway pressure waveform (Paw) occurring after the beginning of each machine breath. Which of the following can explain this finding? A. improper sensitivity setting B. presence of auto-PEEP/air-trapping 464 C. a leak in the patient-ventilator system D. inadequate inspiratory flow setting

A drop in pressure ('scalloping') during volume controlled ventilation indicates inadequate inspiratory flow. Normally, pressure should rise after inspiration begins. To correct this problem increase the inspiratory flow until the "scalloping" of the pressure waveform disappear. The correct answer is: inadequate inspiratory flow setti

An adult CCU patient on a nonrebreathing mask at 12 L/min complains of discomfort and feeling 'closed-in.' The doctor foresees the need to maintain a high FIO2 for at least 12 more hours. Which of the following would you recommend? A. switching to a high flow cannula at 15-20 L/min B. decreasing the nonrebreathing mask flow to 6 L/min C. switching to a 50% air-entrainment/venturi mask D. increasing the nonrebreathing mask flow to 'flush'

A high flow nasal cannula system is a good alternative to a nonrebreathing mask in patients requiring precise moderate to high FIO2s. When delivering 100% O2 to an 420 adult at flows between 15 to 10 L/min, these systems typically provide FIO2s between 0.80 and 0.90, equivalent to the high end provided by a nonrebreathing mask. The correct answer is: switching to a high flow cannula at 15-20 L/min

You are using a portable pressure-cycled ventilator with a heated humidifier and IPPB circuit to temporarily ventilate an intubated patient who is regaining consciousness in the recovery room. When you check the patient and ventilator, you notice that the inspiratory time is prolonged and the machine does not cycle off without active patient effort. How should you correct the problem? A. increase the control pressure B. switch the unit to 100% source gas C. check/adjust ET tube cuff pressure D. decrease the sensitivity setting

A prolonged inspiratory time and delay in off-cycling with a pressure-cycled ventilator indicate a leak. Leakage may be occurring around the ET tube cuff, which can be rectified by adjusting cuff pressure. Alternatively, leakage may be occurring in the delivery circuit. The two most common sources of circuit leakage would be loose connections at the humidifier or (for IPPB circuits) at the nebulizer manifold. The correct answer is: check/adjust ET tube cuff pressure

A patient is receiving appropriate oxygen therapy via a simple mask at 5 L/min but complains that the mask is confining and interferes with eating. Which of the following oxygen-delivery devices is a suitable alternative? A. nasal cannula at 4-5 L/min B. nasal cannula at 2 L/min C. nonrebreather mask at 10 L/min D. a 28% venture mask at 10 L/min

A simple mask with an input flow of 5 L/min usually provides about 35-40% oxygen. A nasal cannula with an input flow of 4-5 L/min would yield an Fio2 of 36-40% and is therefore the best answer. All other choices would result in a substantial change in the Fio2, and the choice involving a mask would not address the issues of confinement and difficulty eating. The correct answer is: nasal cannula at 4-5 L/min

Which of the following is a major component of an typical adult home nasal CPAP apparatus? A. a high pressure gas source B. spring-loaded exhalation valve C. form-fitting nasal mask D. double-limb breathing circuit

A typical home nasal CPAP apparatus consists of a computer-controlled flow generator or blower, single-limb breathing circuit and a fitted nasal or oronasal mask. A humidifier system is optional. Most systems provided pressures in the 2.5 to 20 cm H2O range. Some units have a 'ramp' feature that gradually raises the pressure to the prescribed level over a time interval; others can 'self-adjust' the applied pressure based on the degree of obstruction (auto-titrating CPAP). 376 The correct answer is: form-fitting nasal mask

A respiratory therapist is assisting a physician with a tracheostomy for a patient who is receiving PC ventilation. Following percutaneous placement of a tracheostomy tube, the therapist observes increasing heart rate, decreasing exhaled tidal volume, and increasingly distant breath sounds over the right chest. The therapist should anticipate treatment for: cardiac tamponade. a pneumothorax. a lacerated blood vessel. an anteriorly displaced tub

A. Cardiac tamponade will present with tachycardia and tachypnea, but should not cause a change in the exhaled volume or ventilation to the right lung. (c) B. A pneumothorax will result in tachycardia, decreased ventilation, and decreased breath sounds on the affected side. (u) C. Lacerating a blood vessel may result in hemorrhage; however, it should have no immediate effect on the exhaled volume or diminish breath sounds. (u) D. Anterior displacement of the tube may result in subcutaneous emphysema or airway obstruction.

The predicted FVC value for a African American patient is: A. 10-15% higher than for Caucasians B. the same as for Caucasians C. 20-25% less than for Caucasians D. 10-15% less than for Caucasians

African Americans have been shown to have an FVC that is 10-15% les than age- and height-matched Caucasians. The correct answer is: 10-15% less than for Caucasians

During resuscitation of an adult patient with ventricular fibrillation, sodium bicarbonate should be considered: A. for severe hypoxemia B. if hyperkalemia is present C. if the patient has respiratory acidosis D. immediately before attempting defibrillation

According to the AHA, routine use of sodium bicarbonate is not recommended for patients in cardiac arrest. However, HCO3 is indicated during CPR if (1) hyperkalemia is present, (2) there is a known pre-existing bicarbonate responsive metabolic acidosis, or (3) the patient is suffering from an overdose of tricyclic antidepressants. The correct answer is: if hyperkalemia is present

Which of the following statement about chest compressions during neonatal resuscitation is correct? A. compressions and ventilations should be delivered simultaneously B. the compression to ventilation ratio should be 3:1 C. compressions should exceed 1/2 the AP chest diameter D. compressions should be delivered on the top 1/3 the sternum

According to the American Heart Association (AHA) guidelines for compression and ventilation of neonates, compressions should be delivered on the bottom 1/3 the sternum with 90 compressions and 30 breaths (3:1 ratio) to achieve approximately 120 events per minute. Each event is thus allotted about 1⁄2 second (requiring a compression rate > 100/min between breaths), with exhalation occurring during the first compression after each ventilation. The correct answer is: the compression to ventilation ratio should be 3:1

Which of the following best indicates that a patient's tissues are adequately oxygenated? A. PaO2 of 85 torr B. PvO2 of 30 torr C. SvO2 of 75% D. SaO2 of 90%

An SvO2 of 75% is normal and correlates with normal tissue oxygenation. A PvO2 of 30 torr indicates hypoxemia. Normal ABG values do not necessarily correspond to normal tissue oxygenation values. The correct answer is: SvO2 of 75%

The radiologist remarks upon viewing a 65-year-old patient's PA chest radiograph that the patient has an increased cardiothoracic ratio. This likely indicates that: A. the patient has an athletic heart B. the patient has a left pleural effusion C. the patient has an abnormal heart beat D. the patient has a left middle-lobe infiltrate

An increased C:T ratio of greater than 50% indicates an enlarged heart and is abnormal. The patient should be evaluated for left-ventricular failure. A left pleural effusion is identified by an obscured left costophrenic angle; a pleural air/fluid level may be seen. A lift-middle-lobe infiltrate will have a distinctive shadow over the affected lung area, not the heart. The correct answer is: the patient has an abnormal heart beat

In a neonate, all of the following are indication of HFV except: A. cleft palate before surgical correction B. PPHN C. RDS unresponsive to conventional ventilation D. mechanical ventilation patient with unresolved pneumothorax

An oral or nasal ET tube is inserted into a patient's airway before a cleft palate is surgically repaired; a standard neonatal vent is used if needed. There is no special reason to use HFV. Patients with all of the other clinical situations have been shown to benefit from HFV. The correct answer is: cleft palate before surgical correction

To get a better view of the glottis before inserting an ET tube, you could ask the attending nurse to: A. place downward pressure on the cricoid cartilage B. maximally flex the patient's neck and head C. insert a second laryngoscope in the mouth D. pass a suction catheter through the nasopharynx

Applying downward pressure to the cricoid cartilage (similar to the Sellick maneuver used to prevent aspiration) displaces the larynx posteriorly, which may help align the laryngopharynx and trachea and provides better visualization of the glottic opening, at least in some patients. If cricoid pressure fails to improve or worsens the laryngoscopic view (which also can occur), it should be terminated. The correct answer is: place downward pressure on the cricoid cartilage

In order to assure a stable FIO2 under varying patient demands, an oxygen delivery system must: A. provide all the gas needed by the patient during inspiration B. have a reservoir system at least equal to the tidal volume C. maintain flows at least equal to the patient's peak flows D. automatically change its flow in response to patient demands

As long as a device provides all the inspiratory gas needs of the patient, a stable FIO2 is assured. This need can be met either by providing a volume of gas equal to the inspired volume, or by matching the patient's inspiratory flow with an equal flow of the prescribed mixture. Devices supplying all a patient's inspired gas needs at a given FIO2 are called fixed-performance delivery systems. The correct answer is: provide all the gas needed by the patient during inspiration

A physician's order for a neuromuscular patient being discharged to the home care setting specifies 'IPPB q4h while awake.' The patient has no other respiratory care equipment needs. Which of the following devices would you consider most appropriate for this patient? A. Bird Mark 7 B. Bennett AP-5 C. Bird Mark 8 D. Bennett PR-1

Assuming that the patient has no other respiratory care equipment needs (e.g., supplemental O2, aerosol therapy, etc.), a small electrically powered IPPB device like the Bennett AP-5 should suffice. Pneumatic devices such as the Bennett PR-2 and the Bird Mark 7 and 8 require a 50 psi gas source to operate and thus would needlessly require use (and frequent delivery) of large high pressure gas cylinders. The correct answer is: Bennett AP-5

A 73 year old nursing home patient is hospitalized with chills, fever and a productive cough. A chest X-ray indicates infiltrates in the right upper and middle lobes. Her SpO2 on room air is 85%. Which of the following is most likely to aid in the patient's management? A. O2 therapy titrated to achieve SpO2 > 90% B. incentive spirometry 10 breaths per hour C. bland heated aerosol therapy by mask QID D. postural drainage and percussion QID

Based on her history and symptoms, the patient likely has a community acquired pneumonia (CAP). For patients with CAP the Infectious Diseases Society of America recommends initial empiric antimicrobial therapy until laboratory results can be obtained to guide more specific treatment. Given that the patient also is moderately hypoxemic, O2 therapy should be administered to raise her SpO2 to at least 90%. Postural drainage and percussion are not recommended for the routine treatment of uncomplicated pneumonia. If secretion retention is a problem, early mobilization and directed coughing should be implemented. Positive expiratory pressure (PEP) therapy might also be considered if secretion retention persists. The correct answer is: O2 therapy titrated to achieve SpO2 > 90%

Mechanical ventilation in the home setting can be provided by which of the following methods? A. positive pressure via an intact upper airway B. nasal continuous positive airway pressure C. positive pressure via an oral endotracheal tube D. negative pressure via an oral endotracheal tube

Based on individual evaluation of patient need, one of two major support approaches may be considered for home mechanical ventilation: invasive or noninvasive support. Invasive ventilatory support in the home always involves application of positive pressure ventilation via tracheotomy (NOT via oral or nasal endotracheal tubes). Noninvasive approaches include both positive and negative pressure ventilation via an intact upper airway, diaphragmatic pacing and abdominal displacement methods. Standard nasal CPAP does not augment ventilation and is thus not considered a mode of ventilatory support. 406 The correct answer is: positive pressure via an intact upper airway

A patient with chronic bronchitis is receiving volume controlled A/C ventilation. Wheezing is heard over all lung fields, rhonchial fremitus is felt over the central airways, and secretions are thick. The patient's peak pressure is 45 cm H2O and plateau pressure is 20 cm H2O. Which of the following would be useful to treat the patient's condition? A. triamcinolone (Azmacort) B. cromolyn sodium (Intal) C. pancuronium bromide (Pavulon) 466 D. levalbuterol (Xopenex)

Based on the patient's diagnosis and clinical presentation, the immediate problem is bronchospasm (wheezing) and thick secretions (rhonchial fremitus) causing an increase in airway resistance (increased PIP - plateau). A bronchodilator like levalbuterol AND possibly a mucolytic like acetylcysteine are indicated. Pancuronium is a neuromuscular blocking agent, triamcinolone a corticosteroid and cromolyn a mast cell stabilizer, none of which will aid in bronchodilation or secretion clearance. The correct answer is: levalbuterol (Xopenex

An ICU nurse suspects that her orally intubated patient has partially extubated himself. After confirming significant leakage around the patient's mouth your next step should be to: A. measure the cuff pressure B. push the tube down 2-3 cm C. insert a supraglottic airway D. fully withdraw the tube

Because partial ET tube extubation can mimic a blown cuff, the first step in dealing with this problem is to rule out a large cuff leak by quickly measuring the cuff pressure. If the cuff is not blown, partial extubation is likely, in which case you should deflate the cuff, remove the securing tape, and try to reposition the tube back into the trachea. 354 After repositioning the tube, you should check placement via auscultation, capnography, CO2 colorimetry and/or use of an esophageal detection device or endoscope, followed by a confirming chest X-ray. If this does not reestablish the airway, you may need to remove the ET tube, provide bag-mask ventilation with O2, then consider insertion of a supraglottic airway or reintubation. The correct answer is: measure the cuff pressure

Before connecting the sample syringe to an adult's arterial line stopcock, you would: A. flush the line and stopcock with the IV solution B. aspirate at least 5 mL fluid/blood using a waste syringe 392 C. align the stopcock off to the patient, on to the flush solution D. increase the flush solution bag pressure by 20-30 mm Hg

Before connecting a sample syringe to an adult's arterial line stopcock, you must aspirate at least 5 mL fluid/blood or 5-6 times the tubing volume into a waste syringe, reposition the stopcock handle to close off all ports and disconnect and properly discard the waste syringe. The correct answer is: aspirate at least 5 mL fluid/blood using a waste syringe

In most blood gas analyzers, what media is used to calibrate the pH electrode? A. tonometered whole blood samples B. commercial calibration control media C. precision gas mixtures (O2/CO2) D. standardized buffer solutions

Blood gas analyzers regularly calibrate themselves by adjusting each electrode's output signal when exposed to media having known values. In most units, the media used to calibrate the gas electrodes are precision mixtures of O2 and CO2. For the pH electrode, standard pH buffer solutions are used. The correct answer is: standardized buffer solutions

Continuous positive airway pressure (CPAP) is indicated as a treatment for: A. post-operative atelectasis B. acute exacerbation of COPD C. hypercapnic respiratory failure D. acute pulmonary emboli

CPAP is indicated to treat hypoxemia due to shunting in patients with adequate spontaneous ventilation (e.g., ARDS, IRDS). CPAP is also used to treat patients with atelectasis (intermittent therapy), CHF/pulmonary edema (short term application) and sleep apnea (nocturnal application). CPAP is contraindicated if the patient cannot maintain adequate ventilation on their own, e.g., hypercapnic ventilatory failure, acute exacerbation of COPD. The correct answer is: post-operative atelectasis

A patient with a neurologic disorder exhibits sleep apnea accompanied by Cheyne-Stokes breathing which is not resolved with CPAP. To overcome this problem, you would recommend a trial of A. volume control ventilation (VC) B. adaptive servo ventilation (ASV) C. bi-level positive airway pressure (BiPAP) D. pressure control ventilation (PC)

Cheyne-Stokes respirations consist of regularly repeating periods of central apneas or hypopneas alternating with periods of hyperpnea in a gradual crescendo and 344 decrescendo pattern. This pattern of breathing is often associated with heart failure or neurologic disorders. While CPAP can reduce Cheyne-Stokes respirations, adaptive servo ventilation (ASV) can fully normalize the breathing pattern in most affected patients. ASV devices apply different levels of pressure support. During periods of hypopnea the inspiratory pressure is increased while it is reduced to the lowest possible level during hyperpnea. Note that based on current research, ASV is not recommended for patients with sleep disordered breathing/Cheyne-Stokes breathing associated with symptomatic heart failure with a low ejection fraction. The correct answer is: adaptive servo ventilation (ASV)

Which of the following apnea monitor event recording patterns is most consistent with motion or activity artifact? A. increased respirations, increased heart rate B. decreased respirations, decreased heart rate C. increased respirations, decreased heart rate D. decreased respirations, increased heart rate

Comparison of the impedance signal to the ECG can help identify artifacts associated with patient motion or increased activity. Typically this pattern involves both a marked increase in the amplitude of the impedance signal, in combination with an increase in heart rate. The correct answer is: increased respirations, increased heart rate

To obtain the best measure of whole-body tissue oxygenation in a critically ill patient, you would recommend which of the following? A. a transcutaneous monitor B. a pulmonary artery catheter C. a brachial artery line D. a multi-wavelength pulse oximeter

Currently there are two measures used to assess tissue oxygenation: 1) the mixed venous O2 content or saturation (CvO2 or SvO2) and 2) the central venous O2 content or saturation (CcvO2 or ScvO2). The mixed venous O2 parameters remain the gold standard for assessing whole-body tissue oxygenation, but can only be obtained from the distal port of a pulmonary artery (PA) catheter. With decreased use of PA catheters, clinicians have turned to assessing the central venous parameters, which are more readily (and safely) obtained from a CVP line. Due to placement in the superior vena cava, the ScvO2 reflects brain/upper body tissue oxygenation only, which typically run 2-3% lower that that in true mixed venous blood (due to high brain O2 extraction). The correct answer is: a pulmonary artery catheter

A major problem in applying ribavirin (Virazole) aerosol to children undergoing mechanical ventilation is: A. loss of most of the drug content through evaporation B. denaturation of the drug due to high circuit temperatures C. reconcentration of the drug in the ventilator humidifier D. drug precipitation in the circuit causing valves to jam

Due to precipitation of the drug in the ventilator circuit, ribavirin aerosol can cause obstruction or jamming of the expiratory valve mechanisms. Various circuit and filter adaptations can help overcome this problem. Note that the American Academic of Pediatrics currently recommends against routine use of aerosolized ribavirin in infants or children with bronchiolitis. However, this drug may be helpful in managing patients with severe cases of bronchiolitis or those who are immunocompromised and/or have severe cardiopulmonary disease. Ribavirin also may be helpful in treating RSV infections in immuncompromized adults. The correct answer is: drug precipitation in the circuit causing valves to jam

What is the maximum flow range you can expect to achieve with a standard air entrainment nebulizer set to deliver 40% oxygen? A. 45-60 L/min B. 60-75 L/min C. 75-90 L/min 402 D. 30-45 L/min

Due to the small size of the nebulizer jet, the maximum O2 inflow is restricted to between 12-15 L/min at 50 psig. With a set oxygen concentration of 40% (air/O2 ratio = 3:1), the total output flow will range from about 48 to 60 L/min (12 x 4 to 15 x 4). This may be barely adequate for some patients and inadequate for those with increased peak inspiratory flow rates or high minute ventilations. If higher flows are needed, one could hook two standard nebulizers together in parallel or use a gas injection nebulizer, e.g. a MistyOx. The correct answer is: 45-60 L/min

The normal end-tidal CO2 percentage as measured by capnography ranges between: A. 35-45% B. 3-4% C. 5-6% D. 7-8%

During Phase III (the last portion) of a single breath capnographic tracing, pure alveolar gas is exhaled and the peak or end-tidal CO2 partial pressure (PETCO2) is achieved. PETCO2 levels can be displayed and reported as either a PCO2 or as a percent. A normal PETCO2 ranges between 35-43 mm Hg. At sea level, this equates to about 5-6% CO2. The correct answer is: 5-6%

While assisting a physician who is inserting a pulmonary artery catheter, you note a changeover on the monitor from pulsatile pressures of about 25/5 mmHg to pulsatile pressures of 25/15 mm Hg. Which of the following has occurred? A. the catheter has advanced from right atrium to right ventricle B. the catheter has moved from right ventricle to pulmonary artery C. the catheter has advanced into the pulmonary wedge position D. the catheter has moved from the vena cava into the right atrium

During insertion of a pulmonary artery catheter, pressure waveforms indicate its position. In the vena cava/right atrium (RA), pressures are < 10 mm Hg and barely pulsatile. As the catheter moves into the right ventricle, a changeover to pulsatile pressures of about 25/5 mmHg occurs. As the catheter passes into the pulmonary artery (PA), pulsatile pressures continue, but with a raised diastolic baseline (about 25/15 mm Hg). In the "wedge" position (PAWP), strong pulsations are lost, and pressures drops to 6-12 mm Hg. The correct answer is: the catheter has moved from right ventricle to pulmonary artery

In reviewing the PFT results of a 67 year old smoker with an admitting diagnosis of emphysema and chronic bronchitis you would expect which of the following general findings? A. increased airway resistance and decreased lung compliance B. increased airway resistance and increased lung compliance C. decreased airway resistance and decreased lung compliance D. decreased airway resistance and increased lung compliance

Emphysema and chronic bronchitis are components of chronic obstructive pulmonary diseases (COPD). COPD patients characteristically have increased expiratory airway resistance. If emphysema is present, destruction of elastic tissue in the lung can also INCREASE overall compliance. In fact, emphysema is one of the only pulmonary disorders that is associated with greater than normal lung compliance. The correct answer is: increased airway resistance and increased lung compliance

You would add a corrugated extension tubing to the standard circuit of an inexsufflation (cough assist) device in order to: A. decrease resistance B. trap secretions C. increase deadspace D. trap condensate

Especially when applied to patients with copious secretions, you should use a means for trapping secretions that could accumulate and obstruct the circuit. Standard water traps, sputum traps, or extension tubing with corrugated inner walls can be used for this purpose. The correct answer is: trap secretions

You obtain an expired carbon monoxide (CO) reading of 3 ppm on a patient presented to the emergency room. Based on this finding, you can conclude that the patient: A. can be classified as a nonsmoker B. has abstained for more than 12 hours C. has been exposed to secondhand smoke D. has smoked within the prior 12-24 hours

Expired CO analysis is used primarily to monitor patients' smoking status. Although no universal CO cutoff exists, recent research supports using a reading in the 6 to 8 ppm range to ascertain smoking status. Lower readings indicate a nonsmoking status, with higher readings indicating that smoking has probably taken place in the preceding 12- 24 hours. A higher cutoff (10-11 ppm) should be applied to patients with inflammatory lung diseases such as asthma or COPD because inflammation boosts endogenous CO production. Exposure to secondhand smoke has a negligible effect on expired CO levels. The correct answer is: can be classified as a nonsmoker

When selecting an endotracheal tube, you should consider which of the following to minimize airflow resistance? A. outside diameter (OD) B. cuff compliance C. inside diameter (ID) D. component materials

Flow resistance through an endotracheal tube depends on both the tube's inside diameter (ID) and its length, with the ID being the most important factor. The larger the tube's ID (and the shorter the tube length), the lower its resistance to flow. Neither the outside diameter, component materials of the tube (e.g., PVC vs. rubber) nor the compliance of the cuff affect flow resistance. The correct answer is: inside diameter (ID)

Which of the following is TRUE regarding humidification during high flow nasal cannula therapy? A. an unheated large volume nebulizer with large-bore tubing is required B. a simple unheated wick humidifier provides adequate humidification C. both a heated humidifier and heated delivery circuit are required D. extra humidity is not needed because the upper airway is not bypassed

For O2 delivery, a high flow nasal cannula system typically requires the following equipment: oxygen blender, high flow flowmeter (up to 40 L/min for adults); heated humidifier (or wick-like cartridge); and O2 analyzer. To prevent condensation, the delivery tubing also must be heated, either via wires or coaxial flow of warm water. The correct answer is: both a heated humidifier and heated delivery circuit are required

During properly performed chest compressions on children, the sternum should be compressed: A. about 1/2 inch B. about 1 inch C. about 2 inches D. about 3 inches

For children, you compress the sternum with the heel of one hand 1/3 the AP diameter of the chest or about 2 inches at a rate of at least 100/min. The correct answer is: about 2 inches

A patient receiving CMV with 12 cm H2O PEEP has a corrected VT of 550 ml, a peak pressure of 45 cm H2O and a plateau pressure of 37 cm H2O. What is her compliance? A. 14 ml/cm H2O B. 18 ml/cm H2O C. 22 ml/cm H2O 442 D. 28 ml/cm H2O

For patients receiving ventilatory support, compliance is computed as (where Pplateau is the plateau pressure): (VT - Compressed volume)/(Pplateau - PEEP) With a corrected VT (VT - Compressed volume) of 550 ml, a plateau pressure of 37 cm H2O and 12 cm H2O PEEP, the compliance is: 550/(37-12) = 550/25 = 22 ml/cm H2O. The correct answer is: 22 ml/cm H2O

Which of the following clinical data would help distinguish Guillain-Barré syndrome from myasthenia gravis in a patient with respiratory insufficiency? A. progressive dyspnea B. dysphagia/ loss of gag reflex C. decreased vital capacity D. pain or discomfort to touch

Guillain-Barré (GB) syndrome and myasthenia gravis (MG) are both neuromuscular disorders that if left untreated can progress to cause respiratory insufficiency/failure. Both cause progressive muscle weakness (GB ascending; MA descending), dyspnea (due to impaired diaphragm function) and dysphagia/loss of gag reflex (due to bulbar muscle involvement). In both cases, progressive muscle weakness would manifest as a decrease in vital capacity. Unique to GB is pain or discomfort to touch, a condition called sensory dysesthesia. The correct answer is: pain or discomfort to touch

Which of the following patients is a poor candidate for IPPB therapy? A. a post-op female patient with clinically diagnosed atelectasis B. a chronically hypercapnic patient with full metabolic compensation C. a patient being treated for acute cardiogenic pulmonary edema D. a patient with acute-on-chronic respiratory acidosis

Hypercapnic patients with full metabolic compensation are poor candidates for IPPB therapy. Rapid reduction of the PaCO2 in such patients may impose an acute "metabolic" alkalosis on top of their compensated state. The best supported indications for IPPB are for treating 1) clinically diagnosed atelectasis, 2) impending hypercapnic respiratory failure (acute-on-chronic respiratory acidosis), and 3) 462 decreased compliance in kyphoscoliosis. Note that in most clinical centers BiPAP has largely replaced IPPB for all of these indications except the treatment of atelectasis. The correct answer is: a chronically hypercapnic patient with full metabolic compensation

For which one of the following conditions would you recommend IPPB therapy? A. management of obstructive sleep apnea B. treatment of interstitial pulmonary fibrosis C. management of chronic asthma symptoms D. acute cardiogenic pulmonary edema

IPPB is still used to to treat atelectasis, but has generally been replaced by intermittent CPAP therapy. IPPB may also be used to treat acute cardiogenic pulmonary edema and 343 impending hypercapnic respiratory failure. However, for these purposes IPPB has largely been replaced by noninvasive positive pressure ventilation, e.g. BiPAP. The correct answer is: acute cardiogenic pulmonary edema

A home care patient with obstructive sleep apnea being treated with nasal CPAP complains of chronic nasal congestion. Which of the following would you recommend to overcome this problem? A. recommend switching to a full face mask B. suggest switching to a high flow nasal cannula C. recommend decreasing the CPAP pressure D. recommend a corticosteroid nasal spray

If a CPAP patient complains of nasal congestion due to the therapy, consider adding or adjusting humidification. Alternatively, the patient could use a nasal saline spray or the patient's physician could prescribe a corticosteroid or antihistamine nasal spray to decrease inflammation. If pollen or other airborne allergens are a problem, you should also clean or replace the device's air inlet filter(s). If none of these actions is effective, refer the problem to the patient's physician. Switching to a full face mask will not resolve the problem and likely will be less tolerable for the patient. And although high flow nasal cannulas can create CPAP, the pressure level is not readily determined and the typical system requires high pressure air and O2 gas sources. The correct answer is: recommend a corticosteroid nasal spray

Which of the following should you tell a home care patient to do if her portable O2 concentrator 'Low Battery' indicator activates while on a road trip? A. contact the home care provider B. remove and charge the battery C. switch to 12 volt DC power D. switch to a backup O2 supply

If a portable O2 concentrator Low Battery indicator activates, the patient should immediately switch to the best alternative power source, in this case the 12 volt DC power provided by the vehicle's electric circuit. After providing the unit with an alternative power supply, the battery can be changed or charged. Only if no alternative electrical power is available, should the patient switch to a backup O2 supply. The correct answer is: switch to 12 volt DC power

After insertion of an esophageal-tracheal Combitube® (ETC) in an adult patient in the Emergency Room, you cannot ventilate through either connector. In order to provide effective ventilation to this patient, you should: A. deflate the #1 pharyngeal cuff and ventilate through connector #2 B. withdraw the tube 2-3 cm at a time while ventilating through connector #1 C. remove the ETC and re-establish the airway by any alternative means D. deflate the #2 tracheal/esophageal cuff and ventilate through the connector

If you cannot ventilate through either ETC connectors, the tube likely is inserted too far (causing the large proximal cuff to obstruct the glottis). To correct this problem, withdraw the tube 2-3 cm at a time while ventilating through connector #1 until breath sounds are heard over the lungs. Only If this procedure fails should you remove the ETC and re-establish the airway by an alternative means The correct answer is: withdraw the tube 2-3 cm at a time while ventilating through connector

After removing a patient with an oral endotracheal tube in acute respiratory distress from a ventilator, you find it difficult to provide manual ventilation via bagging. Your next step should be to: A. deflate the ET tube cuff B. remove the bite block C. try to pass a suction catheter D. extubate the patient

In an obstructed airway scenario like this, if you cannot provide adequate ventilation via a bag-valve system, the first thing you should do is try to pass a suction catheter. This may relieve mucous plugging, if that is the problem causing obstruction. The correct answer is: try to pass a suction catheter

The expected PaO2 for an 80 year old man who is otherwise in good health and breathing room air is about: A. 90 torr B. 70 torr C. 80 torr D. 60 torr

In assessing reductions in PaO2, one must take into account the normal decrease in PaO2s that occurs with aging. To estimate the expected PaO2 in older adults we use the following simplified formula: Estimated PaO2 (breathing room air) = 100 - (age/3) Applying this simplified formula to the case at hand, the expected PaO2 would be about 100 - (80/3) = 100 - 27 = 73 torr The correct answer is: 70 torr

In explaining to a terminally ill patient's family the available options for withdrawing ventilatory support from their loved one, which of the following information is essential to share? A. how often you will monitor the patient for distress B. if and when the ET tube will be removed C. how long they can expect the procedure to last D. how much sedation the patient will receive

In explaining to a terminally ill patient's family the available options for withdrawing ventilatory support, you should outline the key steps involved, discuss whether or not the endotracheal tube will be removed, and whether or how supplemental oxygen may be used. You should also be sure the family understands that noisy breathing or gasping can sometimes occur but does not indicate that their loved one is suffering. In addition, all involved personnel should make sure the family knows that the patient's comfort is their primary concern. The correct answer is: if and when the ET tube will be removed

During properly chest compression on infants, the sternum should be compressed: A. 1/3 the AP diameter of the chest B. at least 2 inches (5 cm) C. 1/2 the AP diameter of the chest D. no more than 1/2 inch

In infants, chest compressions should be delivered on the lower third of the sternum to a depth of approximately 1/3 the AP diameter of the chest, using either 2 thumbs with fingers encircling the chest and supporting the back or 2 fingers with a second hand supporting the back. Because it generates higher coronary perfusion pressure than the 2-finger technique, the the 2 thumbencircling technique is recommended for performing chest compressions on new born infants. Compressions and ventilations should be coordinated to avoid simultaneous delivery, with 90 compressions and 30 breaths per minute (3:1 ratio, 120 events per minute). The correct answer is: 1/3 the AP diameter of the chest

The results of which of the following should be used to determine the return of a postoperative patient's ability to breathe adequately after anesthesia? A. mixed venous PO2 B. thoracic gas volume C. maximum inspiratory pressure D. maximal voluntary ventilation

In order to breathe adequately after anesthesia, a patient must be able to generate enough muscular effort to take a deep breath. Of the options listed, only the maximum inspiratory pressure (MIP/NIF) measures muscle effort or force. The correct answer is: maximum inspiratory pressure

A doctor about to follow-up on a patient with moderate COPD after a year in a disease management program asks you advice on what and what not to consider in her evaluation. Which of the following would you recommend NOT evaluating for this patient? A. adequacy of symptom control B. need for long-term O2 therapy C. smoking status D. effects of drug treatments

In order to monitor the success of the self-management process. patients with mild to moderate COPD should be scheduled at least annually for follow-up with their primary provider. Followup should include clinical assessment of smoking status, adequacy of symptom control, presence of complications, effects of drug treatments/inhaler technique, and the need for pulmonary rehabilitation or referral to a specialist. Also at this time objective measures of pulmonary function, BMI and dyspnea should be obtained. The need for long-term O2 therapy is generally assessed only at follow-up for patient with severe COPD.

Pulse oximeter readings are generally unreliable at saturations below: A. 70% B. 80% C. 90% D. 95%

In terms of accuracy, pulse oximetry readings in sick patients usually fall within ±3-5% of those obtained via invasive hemoximetry. In general, the lower the actual SaO2, the less accurate and reliable the SpO2. Most clinicians consider pulse oximeter readings unreliable at saturations below 70%. The correct answer is: 70%

In order to verify the spontaneous rate of breathing of a patient receiving SIMV, you should: A. subtract the preset machine rate from the total displayed rate B. divide the total minute ventilation by the total frequency C. observe actual chest motion and airway pressure deflections D. add the preset machine rate to the total displayed rate

In the IMV mode, the spontaneous breathing rate may be indicated directly on the ventilator, but should always be verified by observing the patient's chest expansion, in 416 conjunction with either the negative deflections occurring on the pressure manometer, or actual recording of spontaneous exhalations. The correct answer is: observe actual chest motion and airway pressure deflections

Which of the following are appropriate instructions for a patient about to receive incentive spirometry? A. exhale maximally after a normal inspiration B. inhale maximally after a normal exhalation C. exhale maximally after a maximally after a maximal inspiration D. inhale maximally after a maximal exhalation

Incentive spirometry is based on a breathing maneuver called the sustained maximal inspiration, or SMI. The SMI mimics a normal sigh by having the patient inhale from the resting expiratory level up to his or her inspiratory capacity (inhaling maximally after a normal exhalation). This increases the transpulmonary pressure gradient and thus alveolar expansion. Moreover, the sustained effort at end-inspiration improves gas distribution to areas of the lung with abnormal time constants. The correct answer is: inhale maximally after a normal exhalation

A patient is receiving oxygen via a 28% air entrainment mask set at the manufacturer's specified input flow of 5 L/min. Which of the following would occur if you were to increase the O2 input flow to 7 L/min? A. the total outflow would increase B. the delivered FIO2 would increase C. the air to oxygen ratio would increase D. the delivered FIO2 would decrease

Increasing the O2 input flow to an air entrainment mask will increase the total outflow flow. However, because the air to O2 ratio remains essentially constant, the delivered O2% remains within 1% to 2% of that specified, regardless of input flow.. The correct answer is: the total outflow would increas

You would recommend oxygen therapy to treat: A. secondary polycythemia 339 B. chronic hypercapnia C. acute myocardial infarction D. absorption atelectasis

Indications for O2 therapy include (1) documented hypoxemia, acute care situations in which hypoxemia is common (e.g., shock, trauma, CO poisoning), acute myocardial infarction (to decrease myocardial workload) and short-term therapy for patients likely to develop hypoxemia (e.g., post-anesthesia recovery). Absorption atelectasis is a potential HAZARD of supplemental O2 therapy. The correct answer is: acute myocardial infarction

Under which of the following conditions would you recommend ending a cardiopulmonary exercise evaluation? A. fall in systolic blood pressure > 20 mm Hg B. heart rate increase from 88/min to 165/min C. 5% decrease in SpO2 from baseline value D. increase in patient's level of dyspnea

Indications for ending a cardiopulmonary exercise evaluation include: (1) significant ECG abnormalities (e.g., dangerous dysrhythmias, ventricular tachycardia, ST-T wave changes); (2) severe O2 desaturation (SaO2 < 80% or SpO2 < 83% and/or a 10% fall from baseline values; (3) a hypotensive response (e.g., a fall of > 20 mm Hg in systolic BP); (4) development of angina; (5) lightheadedness; or 6) a request from the patient to stop the test. The correct answer is: fall in systolic blood pressure > 20 mm Hg

A patient is receiving ventilatory support after thoracic surgery. You observe that the patient is starting to awake and has an FiO2 set at 40% with a PEEP of 5. This tells you that the patient has: A. a need for continued ventilatory support B. a large leak in their endotracheal tube cuff C. a normal maximum inspiratory pressure D. adequate oxygenation to consider weaning

Indications for weaning include: 1) some reversal of the underlying cause of respiratory failure; and 2) adequate oxygenation (P/F > 150-200; PEEP ≤ 8 cm H2O, FIO2 ≤ 0.4-0.5); 3) an arterial pH ≥ 7.25; 4) presence of stable hemodynamics, e.g., no myocardial ischemia or significant hypotension; and 5) presence of spontaneous breathing effort. The correct answer is: adequate oxygenation to consider weaning

In individuals with disorders causing an increased ELASTIC work of breathing, such as pulmonary fibrosis, which of the following breathing patterns results in the minimum work? A. slow and deep breathing B. slow and shallow breathing C. rapid and deep breathing D. rapid and shallow breathing

Individuals with disorders characterized by an increased elastic work of breathing, such as pulmonary fibrosis, tend to assume a rapid and shallow breathing pattern. For these patients, such a pattern results in the minimum mechanical work necessary to effectively ventilate the lungs. The correct answer is: rapid and shallow breathing

Which of the following drugs would NOT be appropriate for the initial management of a child during an acute asthmatic attack? A. albuterol (Proventil) B. cromolyn sodium (Intal) C. terbutaline (Brethaire) D. epinephrine

Initial management of patients suffering from an acute asthma attack always involves administration of a short-acting beta-agonist bronchodilator like albuterol or terbutaline. Cromolyn sodium (Intal) is a controller medication used to prevent asthma attacks. It will do nothing for an acute attack. The correct answer is: cromolyn sodium (Intal)

You obtain the following data on a patient receiving volume control ventilation: Peak pressure 45 cm H2O Plateau pressure 30 cm H2O PEEP 10 cm H2O What is the patient's inspiratory airway resistance? A. 8 cm H2O/L/sec B. 12 cm H2O/L/sec C. 15 cm H2O/L/sec D. 15 cm H2O/L/se

Inspiratory airway resistance (Raw) of a patient receiving volume-cycled ventilation equals the peak pressure (PIP) minus the plateau pressure (Pplat) divided by the inspiratory flow in liters per second, i.e., . Raw = (PIP - Pplat)/flow (L/sec). In this case you must first convert the flow from L/min to L/sec. Flow in L/sec = L/min/60 = 75/60 = 1.25 L/sec. Thus Raw = (45-30)/1.25 = 12 cm H2O/L/sec. Normal Raw for orally intubated patients range from 10 to 15 cm H2O/L/sec; expect higher computed values in patients with airway obstruction. The correct answer is: 12 cm H2O/L/sec

The RT is called to the pediatrics department to help in the evaluation and care of a 4-year-old girl who has been sick with a bad cold for the past two days. In viewing the lateral neck radiograph, you see: 1) clear air column though the upper airway and 2) pointed narrowing of the tracheal air column below the larynx. What is he most likely condition? A. aspirated a coin B. laryngotracheobronchitis C. epiglottitis D. tonsillits

Laryngotracheobronchitis, or croup, is the swelling of the mucous membrane BELOW the vocal cords (subglottic area). This is often identified by a narrowing of the dark air 388 column below the vocal cords. This pointed narrowing is sometimes referred to as the "steeple sign". If she had aspirated a coin, a shape would be seen on the x-ray. Epiglottitis is swelling ABOVE the vocal cords (supraglottic area). The correct answer is: laryngotracheobronchitis

Which of the following devices is LEAST LIKELY to transmit pathogenic organisms to the patient? A. small volume drug nebulizer B. ultrasonic nebulizer C. large volume jet nebulizer D. wick-type humidifier

Liquid nebulizers produce aerosol particles capable of carrying bacteria. On the other hand, wick-type humidifiers do not produce detectable aerosol or transmit bacteria. The correct answer is: wick-type humidifier

An anesthesiologist is planning a rapid sequence induction to intubate an adult trauma patient in the ED. In preparation for intubation, you are ventilating and oxygenating the patient with a bag-valve-mask. The anesthesiologist is concerned about the potential for gastric distention and aspiration before the tube is placed. To help avoid this problem, you would: A. place the patient in the lateral decubitus (side-lying) position B. sit the patient up and keep airway pressure < 20-25 cm H2O C. place gentle pressure on the the patient's upper epigastrium D. rotate the patient's head and neck forty-five degrees to the right

Mask ventilation with O2 for a short period while setting up for rapid sequence intubation provides better protection against arterial desaturation than standard preoxygenation with a nonrebreather. Concerns about increasing aspiration risk due to gastric distension with positive pressure can be addressed by sitting the patient up and by limiting airway pressures during bagging. Gastric distension and regurgitation risk are low if airway pressures are kept below 20-25 cm H2O. The correct answer is: sit the patient up and keep airway pressure < 20-25 cm H2O

Which one of the following measures could be used to evaluate changes in symptoms occurring among participants in a pulmonary rehabilitation program? A. changes in dyspnea scores B. changes in O2 consumption C. changes in blood pressure D. changes in O2 saturation

Measures useful in evaluating changes in symptoms occurring among participants in a pulmonary rehabilitation program include: dyspnea score comparisons (Borg scale); frequency of cough, sputum production or wheezing; weight loss or gain; and standardized psychological tests. Though potentially useful as measures of improvement, O2 consumption, O2 saturation and vital signs are not patient symptoms. The correct answer is: changes in dyspnea scores

You are trying to apply mouth-to-mouth ventilation to an unconscious adult patient, but are unable to maintain a tight seal at the lips. At this point you should: A. place a handkerchief over the victim's mouth and continue B. use the jaw thrust maneuver instead of the head tilt/chin lift C. apply mouth-to-nose ventilation instead of mouth-to-mouth D. immediately apply a series of strong back blows to the victim

Mouth-to-nose ventilation is recommended if ventilation through the victim's mouth is impossible (eg, the mouth is seriously injured), the mouth cannot be opened, the victim is in water, or a mouth-to-mouth seal is difficult to achieve. The correct answer is: apply mouth-to-nose ventilation instead of mouth-to-mouth

Which of the following are prerequisites for successful application of noninvasive positive pressure ventilation (NPPV) in the management of chronic neuromuscular disease? A. intact upper airway function B. MIP > 25 cm H2O (neg) C. normal acid-base balance D. SpO2 > 90% on room air

NPPV is often used in managing the progressive respiratory acidosis that occurs in chronic neuromuscular diseases such as muscular dystrophies, postpolio syndrome and multiple sclerosis. Successful application requires a cooperative and motivated patient with intact upper airway function and minimal secretions. If patients can maintain normal acid-base balance and oxygenation on their own, NPPV probably is not needed. Similarly, a MIP > 25 cm H2O (neg) often is cited as one measure indicating muscle strength sufficient to maintain spontaneous ventilation. The correct answer is: intact upper airway function

High-frequency ventilation would be indicated for all of the following situations except: A. laryngoscopy 458 B. bronchopleural fistula C. near-drowning D. bronchoscopy

Near-drowning patients are ventilated traditionally with AC or SIMV mode with constant volume. HFV has been shown effective ventilating patients with airway procedures such as laryngoscopy and bronchoscopy. HFO has been shown effective in patients with a bronchopleural fistula because the small tidal volumes and low driving pressures do not force gas out through the lung tear. This allow the tissues to heal. The correct answer is: near-drowning

When inspecting the X-ray of a patient with a history congestive heart failure being treated in the Emergency Department, you note patchy densities in the perihilar areas and in the gravity-dependent lower lung fields, with an increased CT ratio. Which of the following is the most likely problem? A. pleural effusion B. bacterial pneumonia C. pulmonary edema D. atelectasis

On X-ray, fluffy or patchy densities in the perihilar areas and in the gravity-dependent lower lung fields is most consistent with pulmonary edema. Cardiomegaly and pleural effusions may also be seen, especially in severe left heart failure. The correct answer is: pulmonary edema

A doctor requests that you increase the expiratory time on a patient receiving volume control ventilation, but not alter the minute ventilation. Which of the following settings would you adjust to fulfill the doctor's request? A. tidal volume B. rate of breathing C. trigger sensitivity D. inspiratory flow

On most ventilators operating in the volume control mode, expiratory time is a resultant value, i.e., not set directly but dependent on other settings. Since the expiratory time is essentially the time left over after inspiration, and since the inspiratory time during volume control is determined by the volume and flow, 465 expiratory time depends on the rate (time factor), volume and flow settings. However, adjusting either the rate or volume will alter the minute ventilation. So in this case, the only way to increase the expiratory time would be to increase the inspiratory flow. Adjusting the sensitivity would only affect the expiratory time if the patient were triggering the ventilator at a rate higher than the set rate (in A/C mode) and you completely 'locked' out breath triggering (pure control mode). Besides potentially causing dyssynchrony, a switch to control mode would also would lower the minute ventilation. The correct answer is: inspiratory flow

You assist a physician insert a pulmonary artery catheter via the subclavian venous route on a patient receiving volume control ventilation in the coronary care unit. Soon thereafter, the ventilator high pressure alarm begins sounding on each breath and the patient appears in acute respiratory distress. Which of the following would you recommend at this time? A. Taking a 12-lead ECG B. Withdrawing the catheter 2-3 cm C. Switching to pressure control D. Obtaining a STAT chest x-ray

One of the complications of pulmonary artery catheter insertion via the subclavian venous route is pneumothorax. The patient's sudden development of respiratory distress and rising airway pressures are consistent with possible development of a tension pneumothorax. Although bedside assessment might indicate this as the cause of the problem, a STAT chest x-ray is needed to confirm it. In the interim, the patient should be ventilated with lower volumes/pressures and— if the situation worsens—a needle thoracostomy or chest tube may have to be inserted. The correct answer is: Obtaining a STAT chest x-ray

A resident in ICU places a pulmonary artery catheter in a mechanically ventilated patient via the subclavian route. 5 minutes later, the attending nurse calls you to the bedside because the ventilator's high pressure alarm is sounding and the patient appears to be rapidly deteriorating. You quickly note decreased breath sounds on the right side and severe hypotension. Which of the following conditions is most consistent with these findings? A. pulmonary edema B. pulmonary embolus C. tension pneumothorax D. gram-negative sepsis

One of the more common complications of pulmonary artery catheter insertion via the subclavian is pneumothorax (suggested by the decreased unilateral breath sounds). In combination with positive pressure ventilation, this can result in a tension pneumothorax. Patients with a tension pneumothorax typically deteriorate rapidly and show signs of cardiovascular collapse, including severe hypotension. This patients likely requires immediate decompression of the tension pneumothorax (via needle thoracostomy), followed by insertion of a chest tube. The correct answer is: tension pneumothorax

Which of the following will have the greatest impact on increasing the mean airway pressure? A. adding 5 cmH2O of PEEP B. increasing the inspiratory flow C. removing 0.5 second of inflation hold D. increasing the expiratory time by 0.25 second

PEEP elevates the patient's baseline pressure and raises mean airway pressure by the same amount. The other 3 options would decrease mean airway pressure by increasing expiratory time. The correct answer is: adding 5 cmH2O of PEEP

The first step in evaluating patients for participation in a pulmonary rehabilitation program is: A. a complete blood gas analysis B. a complete patient history C. a cardiopulmonary stress test D. a pulmonary function test

Patient evaluation should begin with a complete patient history: medical, psychological, vocational, and social. The patient's history should be followed by a complete physical examination. A recent chest film, resting electrocardiogram, complete blood count, serum electrolytes, and urinalysis will provide additional information on the patient's current medical status. The correct answer is: a complete patient history

When inspecting the x-ray of a premature infant in NICU receiving mechanical ventilation, you note a radiolucent band of air surrounding the heart. Which of the following is the most likely problem? A. subcutaneous emphysema B. pneumopericardium C. tension pneumothorax D. pulmonary interstitial emphysema

Pneumopericardium is most commonly seen in patients receiving positive pressure ventilation, especially infants. On the chest x-ray, pneumopericardium may often appears as a 'halo sign" - a radiolucent band of air surrounding the heart. A "continuous diaphragm sign" also may be seen, with air outlining the diaphragm under the heart. The correct answer is: pneumopericardium

Which of the following is a potential hazard of thoracentesis? A. barotrauma B. liver laceration C. pulmonary emboli D. peritonitis

Potential hazards of thoracentesis include bleeding, infection and liver or spleen puncture. In assessing such patients during and after this procedure, you should be mindful of them. Pulmonary emboli, barotrauma and peritonitis generally are not associated with a thoracentesis. The correct answer is: liver laceration

Which of the following ECG leads should be placed in the fourth intercostal space? A. V3 B. V1 C. V5 D. V6

Proper 12-lead placement for the chest leads is as follows: V1: 4th intercostal space, right sternal margin, V2: 4th intercostal space, left sternal margin, V3: Midway between V2 and V4, V4: 5th intercostal space, left midclavicular line, V5: 5th intercostal space, left anterior axillary line, V6: 6th intercostal space, left midaxillary line The correct answer is: V1

Which of the following could be used as quality assurance outcome criteria to assess the effectiveness of airway clearance therapy? A. increase minute ventilation B. change in chest X-ray C. decreased arterial PCO2 D. change in diffusing capacity

Quality assurance outcome criteria to assess the effectiveness of airway clearance therapy could include any of the following: change in sputum production; change in breath sounds of the affected lung fields; changes in the chest X-ray; changes in arterial blood gas values or oxygen saturation; and the patient's subjective response to therapy. The correct answer is: change in chest X-ray

You are setting up transport of an intubated patient who requires frequent endobronchial suctioning to a procedure room that does not have a piped vacuum source. To manage this patient during transport you will need which of the following items: A. a Yankauer suction tip B. a Lukens sputum collection trap C. a battery- or hand-powered suction pump D. a closed suction system

Suctioning requires a negative pressure (vacuum) source. If a central piped DISS vacuum source is not available, for transport a patient needing frequent endobronchial suctioning, you will need a battery- or hand-powered suction pump. The correct answer is: a battery- or hand-powered suction pump

What approximate volume should be used to inflate the larger #1 cuff of an esophageal-tracheal Combitube® (ETC)? A. 12 - 15 mL B. 20 - 30 mL C. 40 - 60 mL D. 80 - 100 mL

The ETC comes in two sizes: 37 and 41 French. Cuff inflation volumes for the 41 Fr: ETC are: large cuff (#1) - 100 mL; small cuff (#2) - 15 mL. For the 37 Fr ETC, you fill the large cuff with 85 mL and the small one with 12 mL. The correct answer is: 80 - 100 mL

An adult patient with a history of COPD and left-ventricular failure has been hospitalized. A series of diagnostic procedures is being performed. The preferred radiographic position to minimize distortion of the heart is: A. anteroposterior B. posteroanterior C. lateral D. oblique

The PA position has the radiographs penetrating the patient from the back to the front. This gives less distortion because the heart is located behind the sternum. The AP view would have more distortion and the heart's shadow would be enlarged on the film. The lateral and oblique views are not helpful in this situation. The correct answer is: posteroanterior

Which of the following would you consider recommending for a home care patient receiving nasal BiPAP who complains of severe nasal dryness? A. room nebulizer B. in-line mesh nebulizer C. heat and moisture exchanger D. in-line water humidifier

The best approach to overcoming nasal drying in a patient using a home BiPAP system would be to place an active water humidifier in-line with the delivery circuit. Depending on the prevailing climate, home humidity levels and patient preferences, the active humidifier may be either heated or unheated. Alternatively, if the problem is intermittent or not severe, use of a saline nasal spray may be sufficient to prevent drying. Due to the leakage that typically occurs in noninvasive circuits, HMEs lose efficiency and thus are not recommended for humidification during this mode of support. although a room nebulizer may increase ambient humidity levels, it represents a potential infection hazard and thus should be avoided. And an in-line mesh nebulizer would be used for drug aerosol deliver, not humidification. The correct answer is: in-line water humidifier

A home care patient with COPD has persistent dyspnea and exhibits signs of increased work of breathing even at rest, but little or no wheezing. The patient currently is receiving no medication. Which of the following drugs would you recommend for this patient? A. salmeterol (Serevent) via MDI 2 puffs bid B. prednisone (Deltasone) 5 mg tab od C. theophylline (Theo-Dur) 150 mg tab bid D. albuterol (Proventil) 0.5 mL via SVN qid

The diagnosis (COPD) and lack of wheezing suggests that the dyspnea is not caused by bronchospasm, but likely due to air-trapping. This would tend to rule out the use of inhaled bronchodilator therapy. On the other hand, slow release theophylline has been shown to decrease dyspnea, air trapping, and the work of breathing in COPD patients, in part by improving the contractility of the diaphragm. These improvements generally occur without any measurable change in pulmonary function. To avoid adverse effects, the drug should be titrated to maintain its therapeutic range of 10-20 mcg/mL. The correct answer is: theophylline (Theo-Dur) 150 mg tab bid

When counseling a patient with sleep apnea on ways to improve sleep, you would recommend which one of the following? A. taking afternoon naps when drowsy B. avoiding heavy exercise before sleep C. keeping the bedroom temperature warm D. regularly taking over-the-counter sleeping pills

The following can interfere with good sleep and should be avoided before bedtime: smoking, alcohol or caffeine intake, heavy exercise, large meals, and spicy foods. In addition, napping during the day should be avoided. If the patient does nap during the day, recommend limiting the duration to less than an hour and avoiding napping after 3:00 PM. Also recommend against misuse of any medications that might affect sleep, including over-the-counter sleeping pills. Keeping the bedroom temperature cool can help create a restful sleep environment. The correct answer is: avoiding heavy exercise before sleep

Auscultation of an 18 year old female patient's chest reveals diffuse wheezing and an irregular heart rate. Chart review indicates a history of frequent nighttime awakenings with dyspnea and cough. Which of the following aerosolized drugs would be most appropriate to administer? 424 A. albuterol (Proventil) B. racemic epinephrine C. acetylcysteine (Mucomyst) D. atropin

The history and presence of wheezing suggests that this patient may well have asthma and is suffering from bronchospasm. The appropriate drug to reverse bronchospasm is a beta-adrenergic like albuterol. Albuterol has minimal beta-1 effects yet significant a beta-2 effect, which relaxes smooth muscle and causes bronchodilation. Racemic epinephrine can cause significant beta-1 stimulation and therefore is generally not appropriate for patients with an arrhythmia. Because asthmatics often have excessive secretions and atropine can significantly dry secretions, atropine is not indicated in this instance. Although acetylcysteine (Mucomyst) can thin secretions, its irritant properties can worsen bronchospasm. The correct answer is: albuterol (Proventil)

An unconscious, apneic adult male patient with a drug overdose has been admitted to the ER. He will be placed onto an older ventilator that cannot compensate for compressed volume. His ideal body weight is 80 kg (176 lb). The most appropriate uncorrected tidal volume would be: A. 800 mL B. 950 mL C. 550 mL D. 400 mL

The initial tidal volume for this patient on an older vent. is 10 mL/kg. This would result is a set, uncorrected VT of 800 mL. Because some volume will be lost, the actual delivered tidal volume will be small than 800 mL and be within the accepted range (6 - 8 mL/kg). The correct answer is: 800 mL

A patient begins breathing and regains effective circulation after resuscitation but remains unconscious. What position should you place this patient in? A. Trendelenburg B. supine C. prone D. lateral recumbent

The lateral recumbent or recovery position (victim placed on side and with the lower arm in front of the body) helps maintain a patent airway and reduces the risk of airway obstruction and aspiration in unresponsive adults with normal breathing and effective circulation. The correct answer is: lateral recumbent

The most common late complication of flexible fiberoptic bronchoscopy (FFB) is which of the following? A. laryngospasm B. pulmonary infection C. bronchospasm D. pneumothorax

The most common late complication of FFB is pulmonary infection, as manifested by fever and the signs and symptoms of pneumonia. Acute complications include damage to the mouth or nose (lacerations and minor bleeding), laryngospasm, transient changes in pulmonary function, hypoxemia, cardiac arrest, and pneumothorax (with transbronchial biopsy). The correct answer is: pulmonary infection

The best site for capillary puncture in an infant is: A. the lateral aspect of the heel's plantar surface B. the anterior curvature of the heel C. the medial aspect of the heel's plantar surface D. the posterior curvature of the heel

The most common site for sampling is the heel, specifically the lateral aspects of the plantar surface. Using this site minimizes the likelihood of nerve or bone damage. The area of the arch should be avoided, as should the posterior medial area; punctures in these areas could cause laceration of the tibial artery or injury to the underlying bone. The correct answer is: the lateral aspect of the heel's plantar surface

Therapeutic gases being delivered to patients need to be humidified because they: A. have low specific gravities B. become less combustible when humidified C. are supplied at low critical temperatures D. are supplied in the anhydrous state

The most common use for humidity therapy is to add water vapor to O2 being administered to patients with normal upper airways. This is necessary because purity standards require medical gases to be in the dry or anhydrous state. For this reason, supplemental humidity may also be provided for anesthetic gases, for gases in the pulmonary function laboratory, or for therapeutic gas mixtures such as oxygen-carbon dioxide or helium-oxygen mixtures. The correct answer is: are supplied in the anhydrous state

While observing a patient's pattern of breathing, you note that the abdomen moves inward as the rib cage expands during inspiration. Which of the following descriptions would you note in the respiratory notes? A. "patient exhibits inspiratory retractions" B. "patient exhibits asynchronous breathing" C. "patient exhibits apneustic breathing" D. "patient exhibits normal breathing pattern"

The pattern of breathing in which the the abdomen moves inward as the rib cage expands during inspiration during inspiration is termed asynchronous breathing or thoracoabdominal paradox. Most commonly, this pattern is seen with respiratory muscle dysfunction or increased work of breathing and can be a sign of impending respiratory failure. Retractions occur when the the skin between or below the ribs (intercostal or subcostal retractions) or above the sternum (suprasternal) is drawn in during inspiration. Retractions are also a sign of increased work of breathing, usually associated with decreased lung compliance or increased airway resistance. Apneustic breathing - refers to a pattern in which every inspiration is followed by a prolonged inspiratory pause, and each expiration is followed by a prolonged expiratory pause (often mistaken for an apnea). This pattern is often caused by damage to the respiratory center in the brainstem. The correct answer is: "patient exhibits asynchronous breathing"

In order to assess the circulatory status of an infant during basic life support, which of the following pulses would you palpate? A. brachial B. ulnar C. carotid D. popliteal

The presence of a pulse is the most reliable indicator of effective chest compressions. In adults and children you should assess the carotid or femoral pulse; in infants and neonates, you can more easily palpate the brachial pulse. The correct answer is: brachial

After inspecting the rhythm strip of a patient under your care in ICU, you note the occurrence of 8-10 premature atrial contractions (PACs) per minute. What action would you recommend at this time? A. administration of an antiarrhythmic drug like lidocaine B. administration of an adrenergic agent like isoproterenol C. administration of an anticholinergic agent like atropine D. cardioversion with a direct current discharge of 100 joules

The primary cause of PACs is an increased excitability of the atrial myocardium. Although strong sympathetic stimulation is a common cause, a variety of chemical or humoral agents may also provoke PACs. Generally, less than six PACs/min requires no treatment; higher rates (above 6/min) are treated with antiarrhythmic drugs like lidocaine. The correct answer is: administration of an antiarrhythmic drug like lidocaine

The primary goal of O2 therapy is to: A. improve tissue perfusion B. decrease the work of breathing C. arterial hypoxemia D. decrease myocardial workload

The primary goal of O2 therapy is to: A. improve tissue perfusion B. decrease the work of breathing C. arterial hypoxemia D. decrease myocardial workload The primary goal of O2 therapy is to prevent or correct arterial hypoxemia/tissue hypoxia. Although O2 therapy can decrease the work of breathing (when hypoxemia causes increased ventilatory demand/respiratory alkalosis) and may decrease heart rate in hypoxemic patients, these are not primary indications. And O2 therapy by itself will not improve tissue perfusion. In fact, high blood PO2s can increase systemic vascular resistance and decrease both cardiac output and coronary blood flow. The correct answer is: arterial hypoxemia

When making routine equipment checks you hear the relief valve of a patient's bubble humidifier sounding. Which of the following is the most likely cause of this problem? A. clogged bubble/diffuser B. delivery tube obstruction C. high wall outlet pressure D. rise in patient ventilation

The relief valve of a humidifier sounds when the pressure in the reservoir container exceeds the valve's threshold pressure (usually between 1-2 psig). The most common reasons for this to occur are 1) an excessive oxygen input flow, and 2) downstream obstruction to outflow. In bubble humidifiers that use a diffuser plate to break up the 403 gas stream, clogging of this element would prevent gas from entering and pressurizing the system. Flowmeter restriction prevents high wall outlet pressures (> 50 psig) from affecting pressure in the humidifier. And changes in patient ventilation would have no effect on pressure in the humidifier. The correct answer is: delivery tube obstruction

A patient has Guillain-Barre syndrome and pneumonia. The patient has just been placed on 35% O2 by mask. The physician asks for your suggestion on the best way to evaluate the patient's overall ability to breathe. What should be recommended? A. doing a full set of pulmonary function tests B. drawing an arterial blood sample for analysis C. performing pulse oximetry D. performing a forced vital capacity measurement

The results of an ABG measurement inform you of the patient's oxygenation status and PaCO2 level. Pulse oximetry gives information only on oxygenation status. PFTs do not give any information on the PaO2 or PaCO2 levels and can be very tiring for the patient. The correct answer is: drawing an arterial blood sample for analysis

To assess gas exchange at the tissues you would sample blood from which of the following? A. systemic artery B. central vein C. pulmonary artery D. peripheral vein

To assess gas exchange at the tissues we need to assess blood after it leaves the capillaries. For the body as a whole, we need to wait until after all the blood from all the capillary beds mixes together, which is complete only in the pulmonary artery. The correct answer is: pulmonary artery

Which of the following should be done in preparation for switching from an esophageal-tracheal Combitube® (ETC) to an oral endotracheal tube? A. make sure the ETC is positioned in the trachea B. avoid aspirating the stomach before the switch C. use the same equipment as for ET tube intubation D. inflate the pharyngeal cuff before laryngoscopy

To switch a patient from an ETC to an oral endotracheal tube, the airway normally must be in the esophagus. If so, gather and prepare all equipment needed for endotracheal intubation, and aspirate any stomach contents through the #2 tracheal/esophageal tube. To intubate while the ETC in place, you must first deflate the large #1 pharyngeal cuff. This will allow for laryngoscopy while the #2 ETC cuff keeps the esophagus occluded. The correct answer is: use the same equipment as for ET tube intubation

A 65-year-old patient with repeated episodes of CHF has a chest radiograph taken. The radiograph shows the left costophrenic angle to be blunted with air/fluid level with a meniscus in the left lower-lobe area. How should this be interpreted? A. pulmonary edema of the left lung B. pneumonia of the left lung C. pleural effusion of the left lung D. pulmonary embolism of the left lung

When you see a blunted costophrenic angle on a chest radiograph, especially for the TMC Exam, you should automatically think pleural effusion. The correct answer is: pleural effusion of the left lung

A patient is receiving continuous flow CPAP at 10 cm H2O. Each time the patient inhales, the pressure decreases to 2 cm H2O. It returns to 10 cm H2O on exhalation. Which of the following is the most likely cause of the problem? A. the flow of gas to the patient system is insufficient B. there is a leak in the patient system C. the patient's endotracheal tube is too small D. the CPAP pressure is set too high for the patient

With any continuous flow system, a drop in pressure below baseline that coincides with the patient's inspiration indicates inadequate flow. In general, a drop in pressure of 2 cm H2O is acceptable; any greater decrease in pressure means that system flow should be increased. The correct answer is: the flow of gas to the patient system is insufficient

The standard frequency for dosing of inhaled tiotropium (Spiriva) is every A. √ 24 hours. B. 4 hours. C. X 12 hours. D. 8 hours.

c) A. Tiotropium is a long-acting anti-cholinergic. The approved FDA time frame between dosage is every 24 hours.

An orally intubated patient is breathing asynchronously with the ventilator. Breath sounds are absent on the left, with dullness to percussion and a left shift of the trachea. Which of the following is the most likely explanation for the problem? A. The patient is experiencing diffuse bronchospasm B. The endotracheal tube is in the right mainstem bronchus C. A tension pneumothorax has developed on the left D. A tracheoesophageal fistula has developed

dull percussion note, tracheal shift toward the affected side and absent breath sounds are all signs of atelectasis. In this instance, slippage of the ET tube into the right mainstem bronchus would likely cause left-sided atelectasis. Diffuse bronchospasm would cause bilateral wheezing and a left-sided tension pneumothorax would cause a hyperresonant percussion note, not a dull one. The correct answer is: The endotracheal tube is in the right mainstem bronchus

Which of the following should a respiratory therapist instruct a patient to use when cleaning the home CPAP mask and connecting tubing? acetic acid hydrogen peroxide dishwashing soap isopropyl alcohol

(c) C. Dishwashing soap is recommended for cleaning home CPAP masks and tubing.

Which of the following is the preferred delivery method for cromolyn sodium to young children? A. MDI with mask B. small volume nebulizer C. MDI with holding chamber D. dry powder inhaler

A small volume nebulizer (SVN) is the method of choice for administering cromolyn sodium to young children. You should use a tightly fitting face mask for any child unable to a mouthpiece. The correct answer is: small volume nebulizer

You would recommend against using noninvasive positive pressure ventilation (NPPV) for a patient with: A. secretions requiring suctioning B. the need for moderate sedation C. facial burns or trauma D. FIO2 needs greater than 40%

Absolute contraindications against using NPPV include the following: need for immediate intubation, hemodynamic instability, active cardiac arrhythmias or ischemia, active upper GI bleeding, uncooperative patient, facial burns or trauma, and the need for airway protection. The correct answer is: facial burns or trauma

Which of the following values are needed to determine a patient's physiologic dead space? mixed expired PCO2 and Pv̄CO2 arterial PCO2 and mixed expired PCO2 Pv̄CO2 and arterial PO2 arterial PO2 and PCO2

B. Physiological dead space is calculated from the Bohr equation [(VD/VT = (PaCO2 - PeCO2)/PaCO2]. Thus, it is necessary to know the values for the arterial and mixed expired values for carbon dioxide.

A doctor orders aerosol therapy for a patient receiving mechanical ventilation who is being provided humidification with a heat and moisture exchanger (HME). To assure effective therapy you must: A. place the aerosol device proximal to the HME in the stream of flow B. remove the HME before aerosol therapy and replace it afterward C. place the aerosol device distal to the HME in the stream of flow D. switch from an HME to an active heated the humidification system

Because an HME traps aerosol, you must you must remove it before aerosol therapy and replace it afterward The correct answer is: remove the HME before aerosol therapy and replace it afterward

Disadvantages of noninvasive ventilation include which one of the following? A. costs more than invasive ventilation B. requires heavy patient sedation C. limits direct access to lower airway D. increases the likelihood of VAP

Disadvantages of noninvasive ventilation (NPPV) include the following: it can only be used in cooperative patients; it does not provide direct airway access (thus increasing the risk of secretion retention), and more therapist time is needed during the initial period of use. On the other hand NPPV may help decrease the incidence of VAP and typically costs less than invasive ventilation. The correct answer is: limits direct access to lower airway

At the bedside of a patient receiving volume control ventilation, you suddenly observe the simultaneous sounding of the high pressure and low volume alarms. Which of following is the most likely cause of this problem? A. a leak in the ET tube cuff B. a mucous plug in the ET tube C. ventilator circuit disconnection D. development of pulmonary edema

During volume-control ventilation, a high pressure/low volume condition signals an obstruction (increased impedance). Although either the mucous plug or the development of pulmonary edema increases impedance, only a plugged ET tube would cause a sudden rise in airway pressure. The correct answer is: a mucous plug in the ET tube

To initiate weaning, a patient was changed from volume control A/C ventilation to pressure support. After 30 minutes on pressure support, the high respiratory rate alarm sounds, with the patient breathing at a rate of 25 to 30 per minute. What change should you make to the ventilator settings? A. increase the pressure support level B. increase the high pressure alarm to 50 cm H2O C. increase the high rate alarm to 30-35 D. switch the patient back to volume control A/C

During weaning procedures, a modest increase in respiratory rate is common and generally should be tolerated up to a maximum of 30-35 breaths per minute. In this instance, the high respiratory rate alarm should be increased to 30-35 breaths per minute. The correct answer is: increase the high rate alarm to 30-35

A 65-year-old female patient has distended external jugular veins even though her head and body are raised 45 degrees above her legs. This would indicate that she likely is: A. hypertensive B. fluid-overloaded 268 C. an emphysema patient D. dehydrated

Fluid overload cause the jugular veins to be distended. Dehydration may result in the jugular veins being flat. Emphysema and hypertension should not have any effect on the jugular veins. The correct answer is: fluid-overloaded

A doctor orders 10 mg of 0.5% albuterol (Proventil) in 50 mL normal saline via continuous nebulization for a patient with asthma. How many mL of albuterol would you place in the nebulizer? A. 0.5 B. 2 C. 5 D. 20

mL = dosage (mg) ¸ concentration (mg/mL) = 10/5 = 2 mL. Note that a 0.5% solution has a concentration of 5 mg/mL. The correct answer is: 2

ARDS patient, what should RT use to evaluate oxygen delivery for optimal PEEP A. ABG B. mixed venous C. serum lactate D. CO

mixed venous

A patient with neuromuscular disease has been receiving ventilatory support for 4 months through a tracheostomy. The patient uses a speaking valve during the day, but receives VC, A/C ventilation at night. Which of the following should be used? tracheostomy button foam cuff tracheostomy tube cuffed tracheostomy tube cuffless tracheostomy tube

u) A. A tracheostomy button will not provide a patent airway for ventilatory support. (u) B. A foam cuff tracheostomy tube is not designed to have the cuff collapsed for prolonged periods of time. A leak in the system may cause a foam cuff to reinflate, blocking the patient's ability to exhale. (c) C. When using a speaking valve, the cuff can be deflated and then reinflated for mechanical ventilation. (h) D. A cuffless tracheostomy tube will not provide a closed system for periods of volume ventilation between ventilatory support.

In which of the following circumstances will tracheal secretions tend to dry in an intubated patient? a water vapor pressure of 47 torr a relative humidity of 100% at 22° C (71.6° F) a dew point of 37° C (98.6° F) an absolute humidity of 44 mg/L

u) A. A water vapor pressure of 47 torr provides 100% humidity at body temperature. (c) B. The absolute humidity at this temperature is inadequate. (u) C. A dew point of 37° C (98.6° F) indicates the gas is completely saturated at that temperature. (u) D. The absolute tracheal humidity must be greater than or equal to 30 mg/L.

A patient lost an unknown quantity of blood as a result of a motor vehicle crash. To fully assess oxygen delivery, a respiratory therapist should recommend: a complete blood count. exhaled nitric oxide. crossmatch of the patient's blood type. serial blood pressure assessment

(c) A. A complete blood count will provide a hemoglobin value that is used to calculate oxygen delivery. (u) B. Exhaled nitric oxide can detect an inflammatory process, but it does not measure oxygen delivery. (u) C. A crossmatch of the patient's blood type should be completed in the event of blood loss; however, it does not provide information about oxygen delivery. (u) D. Serial blood pressure assessment should be completed due to volume loss; however, it does not provide information about oxygen delivery.

A 4-year-old child is seen by a respiratory therapist for a follow-up asthma evaluation. The child denies any dyspnea. The parents report giving the child albuterol by inhaler twice daily. The therapist should: r explain the purpose of the quick-relief medication. demonstrate peak flow monitoring. recommend changing to a small-volume nebulizer. develop an exercise regimen for the child.

(c) A. Albuterol is a quick-relief medication. It should be used as needed, not at a scheduled frequency. (u) B. Peak flow monitoring will not address the need to reeducate the parents on the indications and use of a quick-relief inhaler. (u) C. The route of administration does not need to be addressed. Rather, education on the indications and use of a quick-relief inhaler is warranted. (u) D. An exercise regimen will not address the family's educational needs.

A male patient who is 180 cm (5 ft 11 in) tall and weighs 75 kg (165 lb) is intubated and receiving mechanical ventilation. The endotracheal tube is secured at the 23-cm mark at his incisor. The cuff pressure is 30 mm Hg. Which of the following should a respiratory therapist do? Deflate the cuff until a slight leak is heard at peak inspiration. Reintubate the patient with a larger endotracheal tube. Maintain a cuff pressure of 25 mm Hg. Advance the endotracheal tube to the 25-cm mark.

(c) A. Establishing the minimal leak technique will determine the pressure needed to maintain the tidal volume. (h) B. There is no indication that the tube is too small or that it has been improperly placed. Therefore, reintubation should not be performed. (h) C. Cuff inflation is based on proper protection of the airway, not specific pressure. (h) D. Depth of insertion should be determined by clinical assessment and chest radiograph, not predefined goals.

Which of the following factors will influence the oxygen concentration delivered by a self-inflating manual resuscitator? oxygen flow and reservoir size PEEP setting and oxygen flow bag size and PEEP setting reservoir size and bag size

(c) A. FIO2 is impacted by both the flow into the bag and the presence of a reservoir. With oxygen flow set at 15 LPM and a reservoir attached, the concentration approaches 100%. When a reservoir is not attached, the concentration is approximately 40% less. (u) B. A PEEP valve is a threshold resistor and has no effect on oxygen concentration. (u) C. Bag size has no effect on oxygen concentration. A PEEP valve is a threshold resistor and has no effect on oxygen concentration. (u) D. Bag size has no effect on oxygen concentration.

A 25-year-old patient with apnea is receiving PC ventilation. Arterial blood gas results are as follows: pH 7.20 PCO2 65 torr PO2 70 torr HCO3- 25 mEq/L BE -4 mEq/L A respiratory therapist should recommend increasing the: set inspiratory pressure. expiratory time. sensitivity. peak flow.

(c) A. In PC ventilation, increasing the inspiratory pressure will result in an increased tidal volume and minute ventilation, potentially causing a decrease in the PCO2. (u) B. Increasing expiratory time will decrease tidal volume and minute ventilation, potentially causing an increase in PCO2 and a further decrease in pH. (u) C. Increasing sensitivity will not increase ventilation in this patient. (u) D. Increasing the peak flow in PC ventilation may decrease the tidal volume and minute ventilation, potentially causing an increase in the PCO2.

A patient who is receiving mechanical ventilation requires an FIO2 of 0.70 and a PEEP of 10 cm H2O to maintain an acceptable PaO2. The patient is restless and has become disconnected from the ventilator circuit several times, during which she experiences cardiac rhythm disturbances. A respiratory therapist should conclude that the patient will benefit from a: sedative. mucolytic. neuromuscular blocker. pulmonary vasodilator.

(c) A. Sedation is required to eliminate excessive activity so the patient's cardiopulmonary status can be properly evaluated and treated. (h) B. A mucolytic will thin secretions, which will not correct this patient's problem. (h) C. A neuromuscular blocker will paralyze the patient, but is not recommended without a sedative. (h) D. Decreasing pulmonary vascular resistance will not ease the patient's restlessness.

A fixed-wing medical transport with an unpressurized cabin has ascended to 10,000 ft while transporting a patient with COPD. The patient is receiving nasal oxygen at 2 L/min and becomes agitated and confused. A respiratory therapist should: increase oxygen flow. initiate mask CPAP. recommend a diuretic. recommend a sedative.

(c) A. The patient is experiencing hypoxemia associated with altitude and a lower inspired alveolar PO2. Increasing the oxygen flow will increase the inspired FIO2. (u) B. The patient does not need CPAP. An increase in the FIO2 will help alleviate the hypoxemia associated with the high altitude. (u) C. Administration of a diuretic will delay providing appropriate care. (h) D. A sedative may cause hypoventilation and exacerbate the hypoxemia.

An alert adult patient is receiving CPAP with PS of 5 cm H2O and an FIO2 of 0.28. The patient has a respiratory rate of 25 and a heart rate of 88. Blood gas results are as follows: pH 7.43 PaCO2 35 torr PaO2 95 torr HCO3- 23 mEq/L BE -1 mEq/L Which of the following is a respiratory therapist's most appropriate action? Recommend extubation of the patient. Assess pulmonary mechanics. Wean if the shunt is less than 5%. Maintain current ventilator settings.

(c) A. This patient is alert and ready for extubation. (a) B. Assessing pulmonary mechanics will provide additional clinical information; however, it is unnecessary in this patient. (u) C. The P(A-a)O2 is normal. There are no indications for a shunt study. (u) D. The patient is alert and ready for extubation. Maintaining mechanical ventilation exposes the patient to nosocomial infections and delays discharge.

he most common rhythm in a witnessed sudden cardiac arrest is: ventricular tachycardia. ventricular fibrillation. pulseless electrical activity. asystole.

(c) B. Ventricular fibrillation is the most common cardiac arrhythmia in a witnessed sudden cardiac arrest.

A 34-week gestational age infant has been stable while receiving mechanical ventilation. Oxygen saturation decreases to 85% and does not respond to an increased FIO2. Transillumination of the chest produces a 1-cm halo that extends around the point of contact with the skin. Which of the following should a respiratory therapist recommend? insertion of a chest tube turning the infant to the prone position increased fluid administration additional diagnostic assessments

(h) A. A small halo measuring only 1 cm is normal and does not indicate a pneumothorax. Insertion of a chest tube would be harmful. (u) B. Because increasing the FIO2 had no effect on oxygenation, it is unlikely that changing the infant's position will result in improvement. (h) C. Increased fluid administration will not improve oxygenation and may result in fluid overload which could impair gas exchange. (c) D. Because transillumination findings are normal and increasing the FIO2 fails to improve oxygenation, it is necessary to evaluate other causes that can be contributing to hypoxia.

A 56-year-old man requires continuous mechanical ventilation following cardiac arrest. His heart rate is 110 and blood pressure is 96/50 mm Hg. A pulmonary artery catheter has been inserted. Patient data are below: Body surface area 2 m2 Cardiac output 3.6 L/min Mean PAP 30 mm Hg PCWP 12 mm Hg A respiratory therapist should recommend administering: a beta blocker. a pulmonary vasodilator. a diuretic. an inotropic agent.

(u) A. A beta blocker is used to treat abnormal heart rhythm, hypertension glaucoma, and other conditions. It is not indicated for this patient who is hypotensive. (u) B. A pulmonary vasodilator is not indicated for this patient. (u) C. A diuretic will assist in the formation of urine in the kidneys. This will result in the elimination of salts and water from the body which are used to control hypertension and decrease afterload. (c) D. An inotropic agent will control fluid overload and decrease preload. It should be used to increase the contractility of the myocardium in this patient to increase the BP.

A 50-year-old patient with a tracheostomy is receiving VC ventilation. The high pressure alarm is sounding intermittently and the exhaled tidal volume is reduced. Which of the following should a respiratory therapist do? Change the mode of ventilation. Increase the high pressure alarm setting. Suction the airway. Administer a beta agonist.

(u) A. Changing the mode of ventilation will not resolve the high pressure alarm. (u) B. A high pressure alarm sounds and inspiration ends when a preset pressure is reached. This occurs with increased airway resistance and is most often due to accumulated secretions, which is an indication for suctioning. (c) C. Suctioning the patient will allow delivery of the set tidal volume by decreasing airway resistance. (u) D. There is no indication for administering a beta agonist for a patient who is not experiencing bronchospasm.

A patient is intubated after a motor vehicle crash. A respiratory therapist palpates asymmetrical chest movement during inspiration, but no crepitus. Breath sounds are diminished on the left. Which of the following should the therapist do FIRST? Perform colorimetric capnometry. Administer a bronchodilator. Obtain an arterial blood gas sample. Assess depth of endotracheal tube insertion.

(u) A. Colorimetric capnometry will not address the described problem of asymmetrical chest expansion. (u) B. Relieving bronchoconstriction will not address the described problem of asymmetrical chest expansion. (u) C. Arterial blood gas analysis does not address the described problem of asymmetrical chest expansion. (c) D. Intubation of the right mainstem bronchus most likely explains the asymmetrical chest movement and decreased breath sounds. The best corrective action is to assess the depth of the endotracheal tube insertion.

Which of the following values is used to evaluate an individual's response to inhaled bronchodilators? FVC FEV1 FEF200-1200 diffusing capacity

(u) A. FVC is used to assess the patient's ability to cough and deep breathe. (c) B. FEV1 is the best indicator of reversible airway obstruction in response to inhaled bronchodilators. (u) C. FEF200-1200 is a good indicator of large airway function, but is not useful in evaluating small airway response to bronchodilators. (u) D. Diffusing capacity measures the ability of gases to diffuse across the alveolar-capillary membrane.

Common complications associated with arterial punctures include: fistula formation and hematoma formation. pulmonary embolism and fistula formation. hematoma formation and spasm of the vessel. spasm of the vessel and pulmonary embolism.

(u) A. Fistulas are abnormal connections between tissues or organs and are rarely caused by arterial punctures. (u) B. Fistulas are abnormal connections between tissues or organs and are rarely caused by arterial punctures. A pulmonary embolism is not associated with an arterial puncture. (c) C. Hematomas, or large extravascular blood accumulations, will result when post-puncture pressure has not been applied adequately. Vessel spasm is a common complication associated with needle trauma. (u) D. A pulmonary embolism is not associated with an arterial puncture.

A patient receiving mechanical ventilation had a total fluid intake of 4200 mL and a total fluid output of 1200 mL over a 24-hour period. Which of the following might increase in this situation? lung compliance serum HCO3- P(A-a)O2 hematocrit

(u) A. Fluid output indicates an excess in interstitial fluid which can potentially decrease lung compliance. (u) B. An excess in interstitial fluid will not increase the serum HCO3-. (c) C. An excess in interstitial fluid can impair oxygen diffusion into the capillaries and increase the P(A-a)O2. (u) D. Excessive fluid administration can result in a reduction of hematocrit.

Which of the following indicates a physical conditioning program has been effective for a patient with COPD after 2 months of therapy? Resting pulse rate is unchanged. The 6-minute walk distance is increased by 60 meters. Vital capacity has increased by 5%. FEV1 has improved by 10%.

(u) A. Increased conditioning usually results in a lower resting pulse rate. (c) B. An important part of physical conditioning is an increase in exercise tolerance, easily measured by increased walking distance. (u) C. Pulmonary function results do not normally improve with pulmonary rehabilitation. (a) D. See explanation C.

While performing a patient-ventilator check, a respiratory therapist observes very little condensation in the heated wire circuit. The heated wick humidifier contains an appropriate amount of water. The most likely explanation is that the: minute ventilation is greater than 15 L/min. patient circuit is operating normally. flow is set at too low of a value. room temperature is lower than normal.

(u) A. The minute ventilation will not impact condensation in the circuit. (c) B. The heated wire circuit is designed to maintain gas temperature to prevent condensation. (u) C. Condensation is not significantly affected by low flow. (u) D. A lower than normal room temperature may result in an increase in tubing condensation.

A patient receiving VC, SIMV has the following ventilator settings and blood gas results: FIO2 0.55 Mandatory rate 12 Total rate 12 VT 750 mL pH 7.56 PaCO2 26 torr PaO2 92 torr HCO3- 22 mEq/L SaO2 96% Which of the following should a respiratory therapist recommend? Increase the inspiratory time. Increase the tidal volume to 800 mL. Decrease the FIO2 to 0.50. Decrease the mandatory rate.

(u) A. Increasing the inspiratory time will not affect alveolar ventilation or the respiratory alkalosis. (h) B. Increasing the tidal volume will increase the minute volume and further worsen the respiratory alkalosis. (u) C. The PaO2 is within normal limits. There is no indication to decrease the FIO2. (c) D. Decreasing the mandatory rate will decrease the minute volume and reduce the respiratory alkalosis.

Following placement of a subclavian venous catheter, the high pressure alarm on a patient's ventilator begins sounding. After 10 minutes, the patient's peak inspiratory pressure has increased from 40 to 60 cm H2O and mean arterial pressure decreased from 80 to 40 mm Hg. After ordering a chest radiograph, which of the following should a respiratory therapist recommend FIRST? Increase the peak flow. Administer vasopressors. Perform endotracheal suction. Insert a chest tube.

(u) A. Increasing the peak flow is not indicated when peak pressures are increasing. (u) B. Administering vasopressors will not correct increased airway pressures. (u) C. Accumulated airway secretions can increase inspiratory pressures, but should not cause a decrease in mean arterial pressure. Therefore, suctioning is not indicated. (c) D. Pneumothorax is a possible complication of venous catheter insertion and can result in increased airway pressure. This would sound the high pressure alarm and decrease arterial pressure. Inserting a chest tube is indicated in treating a pneumothorax.

During manual bag-valve ventilation through an endotracheal tube, inadequate ventilation may be caused by: lack of an oxygen reservoir. a deflated endotracheal tube cuff. high pulmonary compliance. disconnection of medical gas.

(u) A. Lack of an oxygen reservoir may affect FIO2, but not delivered volume. (c) B. During manual ventilation, gas will flow past the deflated cuff and result in less volume to the lungs. (u) C. High pulmonary compliance may result in hyperventilation, not inadequate ventilation. (u) D. Manual bag-valve ventilation does not require supplemental medical gas for adequate ventilation.

An adult patient in the ICU is receiving beta-blocker medication and requires bronchodilator therapy. Which of the following should a respiratory therapist recommend? levalbuterol sulfate (Xopenex) albuterol sulfate (Proventil) triamcinolone acetonide (Azmacort) ipratropium bromide (Atrovent)

(u) A. Levalbuterol is a beta-receptor stimulator and may show reduced efficacy in the presence of beta-blocking agents. (u) B. Albuterol is a beta-receptor stimulator and may show reduced efficacy in the presence of beta-blocking agents. (u) C. Triamcinolone acetonide is an anti-inflammatory drug that does not directly achieve bronchodilation. (c) D. Ipratropium bromide is an anticholinergic. Beta blockers do not affect its ability to achieve bronchodilation.

During a ventilator pre-use check to evaluate the integrity of the circuit, a respiratory therapist should assess: pressure during tidal volume delivery. compressible volume of the circuit. peak pressure change when the circuit is capped. volume delivery distal to the exhalation valve.

(u) A. Measurement of pressure will not evaluate the volume delivered or any volume lost due to a leak in the tubing circuit. (u) B. Determining the compressible volume of the circuit does not assess the integrity of the circuit. (c) C. Maintaining peak pressure when the circuit is capped indicates a tight circuit without leaks. (u) D. Although this measures the volume leaving the humidifier, it does not include volume through the circuit and could potentially miss volume lost through any leak distal to the humidifier.

Which of the following may be caused by the administration of aerosolized pentamidine isethionate (NebuPent)? tachycardia bradycardia bronchospasm hypotension

(u) A. No causal relationship between the use of NebuPent and tachycardia has been established. (u) B. No causal relationship between the use of NebuPent and bradycardia has been established. (c) C. Bronchospasm is the most frequently reported adverse effect associated with the use of NebuPent. (u) D. Hypotension is associated with the use of IV or IM pentamidine isethionate (Pentam). It has not been shown to be a problem when the drug is aerosolized.

Which of the following best reflects the adequacy of ventilation? PaO2 r PaCO2 vital capacity FEF25-75%

(u) A. PaO2 is influenced by several factors other than ventilation. (c) B. The arterial level of CO2 reflects the alveolar CO2 and is the best indication of the adequacy of ventilation. (u) C. Vital capacity is a volume measurement of the air within the lung. It is a static volume and cannot reflect the status of ventilation. (u) D. FEF25-75% is a measure of the flow during the midportion of a forced exhalation. It is measured during a single breath; therefore, it cannot reflect the adequacy of ventilation.

An adult patient is receiving PC, SIMV with the following settings: FIO2 0.80 Mandatory rate 15 Spontaneous rate 32 Set inspiratory pressure 20 cm H2O PEEP 10 cm H2O SpO2 92% On inspection, the patient demonstrates suprasternal retractions during spontaneous breaths. Which of the following should a respiratory therapist do NEXT? Decrease rise time setting. Switch to A/C mode. Increase the FIO2 to 0.90. Change inspiratory pressure setting to 25 cm H2O.

(u) A. Rise time is not an active setting. (c) B. The suprasternal retractions are indicative of increased work of breathing and should be resolved by changing the mode to A/C. The respiratory rate may also come down. (h) C. The patient is adequately oxygenated with the current FIO2. (u) D. Increasing inspiratory pressure will not affect the tidal volume of the spontaneous breaths or breathing effort.

During mechanical ventilation, mean airway pressure will always change with a change in: dead space. patient's body position. inspiratory time. FIO2.

(u) A. The addition of dead space to a circuit will increase the degree of CO2 rebreathing, causing the PaCO2 to increase. Mean airway pressure is unaffected. (u) B. Patient body position may or may not alter the mean airway pressure. (c) C. Lengthening the inspiratory time allows the ventilator to increase the tidal volume and the mean airway pressure. (u) D. Mean airway pressure and FIO2 are unrelated.

A respiratory therapist is reviewing a chest radiograph of a patient with a hemothorax and notes the presence of the end of a chest tube. Where should the therapist expect to find the tube as it enters the chest wall? intercostal space line A. second mid-clavicular B. third mid-clavicular C. fourth mid-axillary D. fifth mid-axillary

(u) A. The second intercostal space in the mid-clavicular line is more appropriate for a pneumothorax. (u) B. The third intercostal space in the mid-clavicular is more appropriate for a pneumothorax. (u) C. The fourth intercostal space in the mid-axillary line is too high to drain a hemothorax. (c) D. The fifth intercostal space in the mid-axillary line is appropriate for draining fluid from the chest. A tube placed any higher than the thorax may not adequately drain the fluid.

Which of the following is suggestive of a malfunctioning arterial catheter? patient complaints of pain at the site an increase in the systolic pressure reading the presence of a hematoma difficulty aspirating blood

(u) A. The throbbing sensation may be due to nerve irritation and not related to catheter function. (u) B. The systolic pressure reading would not increase; rather, the pressure tracing would be damped. (u) C. A hematoma is a possible complication of arterial cannulation, but not a sign of catheter malfunction. (c) D. Difficulty withdrawing blood through an arterial catheter is a sign of malfunction.

A 27-year-old male patient has mild asthma. The patient's pulse is 80 before administration of an aerosolized bronchodilator. The patient's pulse rises and stabilizes at 92 during the treatment. Which of the following should a respiratory therapist do? Terminate the treatment and document in the progress notes. Terminate the treatment and notify the head nurse. Continue the treatment as ordered. Continue the treatment with normal saline.

(u) A. There is no indication that the treatment needs to be terminated. (u) B. See explanation A. (c) C. The heart rate response is within normal limits, so continuing the treatment as ordered is appropriate. (u) D. Normal saline has no pharmacological value in this situation.

A 55-year-old male presents with GI bleeding. Arterial blood gas results reveal an arterial oxygen tension of 45 torr. Pulse oximetry readings are unreliable. The patient is in mild respiratory distress, but is not cyanotic. To evaluate the patient's oxygenation status further, it is most important to review the patient's: COHb. Hb. Pv̄O2. P(A-a)O2.

(u) A. There is no indication that this patient was exposed to carbon monoxide. (c) B. Hemoglobin is needed to determine the oxygen-carrying capacity. (a) C. Pv̄O2 may provide additional information regarding oxygenation status, but does not explain the lack of cyanosis. (a) D. P(A-a)O2 may provide additional information regarding oxygenation status, but does not explain the lack of cyanosis.

RT performing a high calibration on a nitric oxide, expected value is A. 45 B. 10 C. 25 D. 80

45

The doctor is concerned that his ARDS patient on pressure control ventilation has high plateau pressures (> 30 cm H2O) and that this may be causing further lung injury. Which of the following modes of ventilation would you consider as an alternative? A. volume control ventilation B. pressure support ventilation C. airway pressure release ventilation D. continuous positive airway pressure

Airway pressure release ventilation (APRV) is a good option to consider in patients with ARDS, especially when the plateau pressures needed to provide adequate ventilation exceed 30 cm H2O. APRV is also indicated to treat refractory hypoxemia due to collapsed alveoli and massive atelectasis. The correct answer is: airway pressure release ventilation

A 45-year-old patient with asthma is prescribed 0.3 mL of albuterol (Proventil) in 3 mL normal saline via small volume nebulizer. Before initiating therapy, you note from chart review that the patient is severely hypertensive and has been experiencing episodes of superventricular tachycardia. You should do which of the following? A. administer the treatment as ordered B. postpone the treatment and consult the physician C. dilute the albuterol with extra normal saline D. decrease the amount of albuterol administered

Albuterol is a beta-adrenergic drug that can increase heart rate and blood pressure. For this reason, hypertension and tachycardia are contraindications to its administration. Whenever a contraindication exists to drug administration, you normally should postpone the treatment and consult with the ordering physician. The correct answer is: postpone the treatment and consult the physician

Which of the following categories of patients are good candidates for negative pressure ventilation? A. patients with acute obstructive disorders of the upper airway B. patients suffering acute exacerbations of chronic lung disease C. patients with end-stage chronic obstructive pulmonary disease D. patients with chronic neuromuscular disorders and normal airways

Although its use has declined in recent years, negative pressure ventilation remains a viable alternative to the positive pressure approach with certain patient categories. Specifically, patients with permanent neuromuscular impairments who retain adequate upper airway protective and clearance reflexes (thereby not needing an artificial airway) but cannot tolerate masks/mouthpieces are ideally suited to ventilatory support via negative pressure ventilation, especially for use in the home. The correct answer is: patients with chronic neuromuscular disorders and normal airways

A patient with asthma is given an adrenergic bronchodilator agent to combat an acute airway obstruction. Instead of demonstrating improvement in airflow, the patient's symptoms worsen (e.g., increased wheezing, etc.). A possible explanation for this observation is: A. tachyphylaxis or tolerance to the agent B. alterations in the V/Q ratio (a beta-2 effect) C. the additive effect of other drug agents D. a paradoxical response to the agent

Although rare, some patients exhibit a paradoxical response to adrenergic bronchodilators in which the symptoms of acute airway obstruction actually are made worse by drug agent. It is believed that this adverse effect is a result of an allergy to some of the metabolic products of the adrenergic drugs. The correct answer is: a paradoxical response to the agent

An 80 kg (176 lb) patient is receiving volume control A/C ventilation with 35% oxygen at a rate of 12/min and a VT of 600 mL. The following information is available (blood gases obtained while on ventilator): pH 7.38 Spon VT 175 mL PaCO2 35 mmHg Spon VE 7.0 L/min HCO3 21 mEq/L VC 600 mL PaO2 110 mm Hg MIP/NIF -10 cmH2O Which of the following actions would be appropriate at this time? A. place the patient on a 40% T-tube and monitor closely B. maintain current ventilator settings and re-evaluate later C. switch the patient to SIMV at a rate of 4/minute D. place the patient on 5 cm H2O CPAP and monitor closely

Although the patient's on-ventilator blood gases are acceptable, the bedside measurements indicate that the patient is not yet ready for a spontaneous breathing trial. With a spontaneous VE of 7.0 L/min and VT of 175 mL, the patient's spontaneous rate is 40/min, with the rapid shallow breathing index (RSBI) over 200 (40/.175 = 228). In general, a RSBI over 100-105 predicts weaning failure, so in this case it would be best to maintain the current ventilator settings and re-evaluate the patient later. The correct answer is: maintain current ventilator settings and re-evaluate later

In individuals with disorders characterized by an increase in airway resistance, such as emphysema, which of the following breathing patterns results in the minimum work? A. deep breathing B. slow breathing C. shallow breathing D. rapid breathing

An increase in airway resistance increases the frictional work of breathing, i.e. the pressure difference due to air flow. Decreasing the rate of breathing will decrease the pressure difference due to air flow. Thus in these patients a slow breathing pattern (decreased flows) will result in the minimum work. The correct answer is: slow breathing

Any sudden occurrence of pulmonary or cardiac distress in older, bed-ridden patients and those having undergone extensive abdominal or pelvic surgery suggest a diagnosis of: A. coronary artery disease B. pulmonary thromboembolism C. anaphylatic shock D. acute left ventricular failure

Any sudden occurrence of cardiopulmonary distress in older, bed-ridden patients and those having undergone extensive abdominal or pelvic surgery should immediately suggest pulmonary thromboembolism. The correct answer is: pulmonary thromboembolism

You would consider modifying any head-down positions used for postural drainage for a patient with: A. cystic fibrosis B. ciliary dyskinesia C. bronchiectasis D. orthopnea

As part of patient assessment prior to instituting postural drainage, you should determine the potential need for modification of the position(s) chosen. Modification of head-down positions may be needed in patients with unstable cardiovascular status, hypertension, cerebrovascular disorders, and orthopnea. Unless accompanied by one of these conditions, cystic fibrosis, bronchiectasis and ciliary dyskinesia are all potential indications for postural drainage, often including head-down positioning. The correct answer is: orthopnea

A patient has been on a high-flow nasal cannula for 2 days following abdominal surgery. The patient's atelectasis has improved and the arterial blood gas results on a flow of 20 L/min and FIO2 of 0.70 (via blender) are as follows: pH 7.37 PaCO2 43 torr PaO2 155 torr SaO2 99% HCO3 24 mEq/L BE +2 Which of the following would you recommend? A. Decreasing the flow B. Decreasing the flow and FIO2 together C. Decreasing the FIO2 D. Changing to standard nasal cannula

Assuming normal Hb and Hct, a PaO2 of 155 torr is excessive and should be lowered. Moreover, after three days on a high FIO2, O2 toxicity should be a concern. So the best action is in this case would be to lower the FIO2, while maintaining the flow at 20 L/min. Only at flows of at least 20-30 L/min can you expect a high flow nasal cannula to deliver the set FIO2 to adult patients. Lowering the flow will have a variable effect on the FIO2 and whenever possible you should try to avoid changing two parameters at the same time. The correct answer is: Decreasing the FIO2

You need to perform nasotracheal suctioning on a patient with retained secretions. As compared to suctioning via a tracheal airway, which of the following complications are unique to this procedure? A. hypotension B. gagging/aspiration C. hypoxemia D. increased ICP

Complication/hazards common to both tracheobronchial and nasotracheal suctioning include hypoxemia, cardiac dysrhythmias, bradycardia, hyper-/hypotension, bronchospasm, atelectasis, increased intracranial pressure and the potential for contamination/infection. Unique complications of nasotracheal suctioning include nasal trauma/epistaxis, pharyngeal trauma, gagging (with potential 241 vomiting/aspiration), and laryngospasm. Also misdirection of the catheter is more common with nasotracheal suctioning.

A trauma patient has been receiving volume controlled SIMV via an endotracheal tube for two days. The attending surgeon anticipates that the patient will likely be on the ventilator for another 3 to 4 days. Which of the following actions would you recommend? A. switch from ET intubation to tracheostomy B. switch to noninvasive positive pressure ventilation C. switch to pressure control ventilation (PSV) D. maintain the endotracheal tube in place

For a patient who has had an endotracheal tube in place for 1-3 days and for whom extubation is anticipated within a week, it is normally best to continue support of the patient with the ET tube in place (not perform tracheostomy). The correct answer is: maintain the endotracheal tube in place

Which of the following is the appropriate load to establish for patients receiving inspiratory muscle training? A. at least 33% of the predicted inspiratory capacity (IC) B. at least 10-15 ml/kg of predicted body weight (PBW) C. at least 30% of the maximum inspiratory pressure (MIP/NIF) D. at least -25 cm H2O, as measured by a calibrated manometer

For inspiratory training to be effective, the load against which the patient breathes must be sufficient to increase muscle strength. The minimal resistance load to achieve this end is an inspiratory pressure that is at least 30% of the MIP/NIF. The correct answer is: at least 30% of the maximum inspiratory pressure (MIP/NIF)

Which of the following drug would you recommend for a patient with acute bronchospasm? A. racemic epinephrine B. acetylcysteine (Mucomyst) C. albuterol (Proventil) D. cromolyn sodium (lntal)

For patients with acute bronchospasm, you normally select an aerosolized bronchodilator (a beta-adrenergic like albuterol and/or a cholinergic blocker like ipratropium). Acetylcysteine is a mucolytic that would only worsen bronchospasm in sensitive patients. Cromolyn is a prophylactic anti-inflammatory useful in preventing bronchospasm in asthmatics. Racemic epinephrine (strong alpha-adrenergic) is selected to reverse the vasodilation and mucosal edema that can cause upper airway obstruction episodes such as croup. The correct answer is: albuterol (Proventil)

In the management of a patient with brain trauma, therapeutic hyperventilation should be used: A. only during the initial 24 hours of management B. prophylactically to prevent an increase in ICP C. in urgent situations such as brain herniation D. to help wean the patient off ventilatory support

For patients with traumatic brain injury, therapeutic hyperventilation should be avoided during the first 24 hours after injury (typically the period with the lowest cerebral blood flow). Hyperventilation should be considered only in emergent situations in which there are signs of brain herniation and/or when other treatment strategies have failed to lower ICP. Hyperventilation should not be used prophylactically. The correct answer is: in urgent situations such as brain herniation

A doctor orders aerosol drug therapy via small volume nebulizer for a patient receiving mechanical ventilation via a dual-limb breathing circuit. To prevent drug residue from affecting ventilator performance you must make sure that: A. both inspiratory and expiratory HEPA filters are in place B. a HEPA filter is in place on the inspiratory limb of the circuit C. a heat and moisture exchanger is in place at the patient connector D. a HEPA filter is in place on the expiratory limb of the circuit

HEPA filters are needed to prevent drug residue from entering the ventilator and affecting its performance. For dual-limb circuits, be sure that inspiratory and expiratory HEPA filters are in place. For single-limb circuits, you normally only need an inspiratory HEPA filter; expiratory filtration may be required on patients with disorders requiring droplet or respiratory precautions. The correct answer is: both inspiratory and expiratory HEPA filters are in place

A cooperative patient receiving aerosol therapy with 0.9% NaCl is unable to produce an acceptable volume of sputum for laboratory studies. The most appropriate action would be to A. administer the aerosol continuously B. change to hypertonic saline C. initiate chest physiotherapy D. perform nasotracheal suctioning

Hypertonic saline solution can help draw fluid out of the airway mucosa. For this reason, the aerosolization of hypertonic saline solution can help mobilize secretions and can be effective with sputum induction. The correct answer is: change to hypertonic saline

A 58-year-old patient in the open heart unit had cardiopulmonary bypass surgery with significant blood loss. On physical exam the patient presents with tachypnea and tachycardia and the SpO2 is 85% on 4 L/min nasal cannula. You should now recommend: A. non-rebreathing mask at 12 L/min B. CPAP with 5 cm H2O pressure C. nasal cannula at 6 L/min D. 60% air-entrainment mask

In combination, the tachypnea and tachycardia and the SpO2 all signal significant hypoxemia (an SpO2 of 85% equates to a PaO2 of 50 torr), compounded by diminished O2 carrying capacity due to the blood loss during surgery. To treat this hypoxemia, you should deliver the highest possible FIO2 to this patient. Among the available options, only the non-rebreathing mask is capable of consistently providing oxygen concentrations above 60%. The correct answer is: non-rebreathing mask at 12 L/min

While given an adrenergic aerosol bronchodilator treatment to an adult patient, you notice an increase in pulse rate from 85/min (pre-treatment) to 125/min. Which of the following would be the correct action in this case? A. terminate the treatment and record adverse response in chart B. use half the standard dosage listed in the package insert C. add more diluent (e.g., saline) to the nebulizer chamber 296 D. temporarily stop the treatment and let the patient calm down

In general, an adrenergic bronchodilator should not be given if a patient already is experiencing tachycardia. Moreover, once a treatment is begun with an adrenergic agent, it should be terminated if the pulse rate increases more than 20 beats/min. The correct answer is: terminate the treatment and record adverse response in chart

The most common problem encountered in applying assist-control mode ventilation is: A. hypoventilation/hypercapnia B. need for neuromuscular paralysis C. hyperventilation/hypocapnia D. increased work of breathing

In the assist-control (A/C) mode patients can trigger machine breaths at a higher rate than the ventilator rate setting. Although this generally increases tolerance and 252 comfort, some patients will develop tachypnea and thus be prone to hyperventilation/hypocapnia due to an excessive minute ventilation. This problem is most common in patients suffering from pain, anxiety and other causes of acute distress. Tachypnea in the A/C mode also can cause air-trapping and auto-PEEP, especially in those with obstructive disorders. The correct answer is: hyperventilation/hypocapnia

Venous return is LEAST impaired by which of the following modes of mechanical ventilation? A. A/C with a mandatory rate of 10/min, 5 cm H2O PEEP B. SIMV with a mandatory rate of 6/min, no PEEP C. SIMV with a mandatory rate of 12/min, 5 cm H2O PEEP D. A/C with a mandatory rate of 15/min, no PEEP

In this instance, the absence of PEEP and the lowest number of "machine" breaths would result in the lowest mean airway pressure, thus resulting in the least impairment to venous return. The correct answer is: SIMV with a mandatory rate of 6/min, no PEEP

A physician orders 2.5 mL ipratropium bromide (Atrovent) 0.2% TID for a COPD patient with recurrent bronchospasm. Which of the following methods would you use to deliver this drug? A. small volume nebulizer with mask B. ultrasonic nebulizer with mask C. small volume nebulizer with mouthpiece D. MDI

Ipratropium bromide aerosol can cause temporary blurring of vision as well as narrow angle glaucoma or eye pain if the solution comes into direct contact with the eyes. Use of a nebulizer with a mouthpiece (rather than face mask) reduces the likelihood of the nebulizer solution reaching the eyes. The correct answer is: small volume nebulizer with mouthpiece

Which of the following are acceptable changes in patient status during a spontaneous breathing trial for weaning from mechanical ventilation? A. heart rate rises from 103 to 118/min B. SpO2 falls from 90% to 80% C. scalene muscle activity increases D. systolic BP falls from 110 to 75 mm Hg

Measures indicating a successful SBT include acceptable gas exchange (SpO2 ≥ 85-90% or PaO2 ≥ 50-60 torr; pH ≥ 7.30; increase in PaCO2 ≤ 10 torr); stable hemodynamics (heart rate <120-140/min; %change < 20%; systolic BP < 180-200 mm Hg and > 90 mm Hg with %change < 20%); and a stable ventilatory pattern (respiratory rate ≤ 30- 35/min, %change < 50%; no accessory muscle use or thoracoabdominal paradox). The correct answer is: heart rate rises from 103 to 118/min

When monitoring a patient during a spontaneous breathing trial (SBT), which of the following observations would cause you to stop the trial and return the patient to ventilatory support? A. decrease in O2 saturation from 91% to 82% B. increase in respiratory rate from 18 to 28/min C. increase in arterial PCO2 from 45 to 53 torr D. increase in heart rate from 98/min to 115/min

Measures indicating failure of a SBT include inadequate gas exchange (SpO2 ≤ 85-90% or PaO2 ≤ 50-60 torr; pH ≤ 7.30; increase in PaCO2 ≥ 10 torr); unstable hemodynamics (heart rate > 120-140/min; %change > 20%; systolic BP > 180-200 mm Hg or < 90 mm Hg or %change > 20%); and an unstable ventilatory pattern (respiratory rate ≥ 30- 35/min or %change > 50%; presence of accessory muscle use or thoracoabdominal paradox). The correct answer is: decrease in O2 saturation from 91% to 82%

A patient with a neuromuscular disorder who is breathing through an intact upper airway is receiving external (vest) oscillation to help mobilize secretions into the large airways, but is having difficulty clearing them. Which of the following techniques would you recommend to facilitate clearance of this patient secretions? A. manually assisted ("quad") cough B. postural drainage, percussion and vibration C. positive expiratory pressure (PEP) therapy D. acetylcysteine (Mucomyst) via aerosol

Patients with neuromuscular disorders can have trouble mobilizing and clearing secretions. To help mobilize secretion in patients with a neuromuscular disorder and an intact upper airway either postural drainage with percussion and vibration or external (vest) or internal (IPV) oscillation are indicated. Once mobilized, the secretions need to be cleared using one or more of the following methods: manually assisted cough, mechanical insufflation-exsufflation or nasotracheal suctioning. In these patients PEP therapy is not effective due to inadequate respiratory muscle strength (needed to generate the expiratory pressure). And acetylcysteine (Mucomyst) via aerosol may help thin/mobilize secretions, but does little to actually clear them from the large airways. The correct answer is: manually assisted ("quad") cough

Which of the following indicates a deficit in fluid balance A. Pedal edema B. Poor skin turgor C. cap refill D. JVD

Poor skin turgor

A doctor wants an outpatient with idiopathic pulmonary hypertension to self administer an inhaled pulmonary vasodilator. Which of the following drugs would you recommend? A. epoprostenol (Flolan) B. diltiazem (Cardizem) C. iloprost (Ventavis) D. bosentan (Tracleer)

Pulmonary vasodilators administered via the inhalation route currently include three prostacyclins: epoprostenol (Flolan), treprostinil (Tyvaso) and iloprost (Ventavis). Only treprostinil (Tyvaso) and iloprost (Ventavis) are approved for administration via the inhalation route, typically for use by outpatients. Inhaled epoprostenol (Flolan) is used off-label in the acute care setting as an alternative to inhaled nitric oxide therapy and typically administered via continuous nebulization using an IV drip into a jet nebulizer. Both diltiazem (Cardizem; a calcium channel blocker) and bosentan (Tracleer; an endothelin receptor antagonist) are used to treat pulmonary hypertension, but are administered orally. The correct answer is: iloprost (Ventavis)

A doctor orders a spontaneous breathing trial (SBT) for a 80 kg (176 lb) male patient intubated with a 6.5 mm endotracheal tube. Which of the following weaning strategies would you recommend? A. CPAP B. simple T-tube C. pressure support D. SIMV

SBTs can be conducted via simple T-tube or via 'on-ventilator' provision of CPAP and/or pressure support. Because this patient's endotracheal tube is smaller than normal for his size, the SBT should be implemented with pressure support, which can help overcome the extra work imposed by breathing through an artificial airway. Pressure support also can boost the VT and help keep respiratory rates in a manageable range. CPAP would be a consideration if either maintaining the patient's FRC or the occurrence of auto-PEEP were at issue. SIMV is not used to conduct spontaneous breathing trials. The correct answer is: pressure support

Which of the following is true regarding synchronous intermittent mandatory ventilation (SIMV)? A. machine breaths cannot be pressure controlled B. asynchrony is prevented during machine breaths C. only partial ventilatory support can be provided D. patient normally contributes to minute ventilation

SIMV allows spontaneous breathing between machine breaths, so that the patient can control both the overall rate and pattern and contribute to the total minute ventilation. SIMV provides full ventilatory support at normal rates and partial support at lower rates. Machine breaths may target either volume (VC, SIMV) or pressure (PC, SIMV) and spontaneous breaths may be pressure supported. Asynchronous breathing still can occur during machine breaths, usually due to improper machine sensitivity or flow settings. The correct answer is: patient normally contributes to minute ventilation

A 65-year-old male patient with COPD is in the emergency department is receiving oxygen therapy by nasal cannula at 6 L/min, with an SpO2 of 98%. He was talkative on admission, but is becoming sleepy after 1 hour of O2 therapy. The most appropriate action is to: A. discontinue the O2 therapy B. switch to a 24% air-entrainment mask C. decrease the cannula flow to 5 L/min D. switch to a nonrebreathing mask @ 10 L/min

Significant increases in blood PaO2 levels in chronic CO2 retainers given moderate to high FIO2s (as here) can cause O2-induced hypercapnia, especially during acute exacerbations. This rise in PaCO2 can diminish the level of consciousness. To avoid this problem, controlled O2 therapy at a low FIO2--such as that provided by a 24% via air entrainment mask--is indicated, along with with careful monitoring of the patient. The correct answer is: switch to a 24% air-entrainment mask

At follow-up in a disease management program, a patient with severe (Stage IV) COPD is diagnosed with pulmonary hypertension and cor pulmonale. Which of the following would you recommend the doctor consider for this patient? A. long-term O2 therapy B. oral corticosteroids C. systemic vasodilators 320 D. a long-acting beta agonist

Stage IV/Severe COPD is often characterized by the development of chronic hypoxemia and its associated disorders, pulmonary hypertension and cor pulmonale. Long-term oxygen therapy (LTOT) has proven effective in treating chronic hypoxemia in these patients, while at the same time reducing the incidence of cardiopulmonary complications and increasing survival The correct answer is: long-term O2 therapy

The nurse calls you over to examine the arterial blood gas results of a 50-kg (110 lb) patient who is receiving volume control A/C ventilation. Currently the patient has a VT of 400 mL, rate of 10/min, and 35% O2. Her blood gas results are as follows: pH 7.30 PaCO2 48 torr HCO3 23 mEq/L BE -2 mEq/L PaO2 73 torr SaO2 94% Based on these values, which of the following changes is appropriate? A. increase the VT B. increase the FIO2 313 C. mantain settings D. increase the set rate

The ABG indicates an acute (uncompensated) respiratory acidosis with satisfactory oxygenation. To correct this, the PCO2 should be reduced by increasing the minute volume. You could increase the minute volume by either increasing the respiratory rate or the tidal volume. However, given that the VT is already about right for this patient (400/50 = 8 mL/kg), it would probably be safer to increase the rate. The correct answer is: increase the set rate

COPD patient has the following arterial blood gas results in room air: pH 7.33 PaCO2 65 torr PaO2 49 torr HCO3 33 mEq/L BE +7 The patient complains of shortness of breath and lightheadedness, has bilateral bronchial breath sounds with inspiratory crackles and exhibits cyanosis around his lips. Which of the following would you recommend for this patient? A. 10 L/min nonrebreathing mask B. 28% air-entrainment mask C. albuterol (Proventil) by SVN D. 4 L/min nasal cannula

The ABG indicates partially compensated respiratory acidosis with severe hypoxemia, likely due to an acute exacerbation of the condition, possibly caused by pneumonia. Given the severity of the hypoxemia, immediate O2 therapy is indicated. However especially during acute exacerbations of COPD, it is recommended that blood oxygen levels be carefully titrated in order to avoid oxygen-induced hypercapnia. Commonly cited target levels are PaO2s in the 55-70 torr range or SpO2s in the 88 to 93% range. Given the added issue of dyspnea, this patient should be placed on a high flow device capable of giving a precise low FIO2, i.e. a 28% air-entrainment device. The correct answer is: 28% air-entrainment mask

Which of the following ventilator graphics displays would be the best choice to assess the work of breathing associated with patient triggering? A. volume vs. time display B. flow vs. volume display C. flow vs. time display D. pressure vs. volume display

The best choice to assess the work of breathing associated with patient triggering using ventilator graphics would be a pressure vs. volume loop. The correct answer is: pressure vs. volume display

Your patient is receiving aerosolized bronchodilators to treat her asthma. What is the best way to determine whether this treatment is achieving the desired goal? A. measure the patient's MIP before-and-after treatment 279 B. measure the patient's inspiratory capacity before-and-after treatment C. measure the patient's FEV1% before-and-after treatment D. calculate the patient's alveolar minute volume

The best way to determine the effectiveness of bronchodilator therapy at the bedside is to measure the patient's forced expiratory flows before and after treatment. Either the FEV1% or the peak expiratory flow rate (PEFR) can be used, although the FEV1% is a more reliable and valid measure. In general, an improvement of at least 12-15% between the pre- and post-test values is needed to indicate reversibility of the obstruction with the bronchodilator. The correct answer is: measure the patient's FEV1% before-and-after treatment

On a patient receiving volume control AC ventilation, you observe a flow-volume with a sawtooth pattern on exhalation. Which of the following actions would you consider most appropriate? A. recommend administering a bronchodilator B. assess the patient's need for suctioning C. measure the endotracheal tube cuff pressure D. switch to pressure control ventilation

The flow-volume loop reveals irregular sawtooth- like oscillations, primarily in the expiratory portion of the loop. This indicates either 1) accumulation of airway secretions in the trachea/large airways (requiring suctioning), or 2) condensate partially blocking the expiratory limb of the ventilator circuit proximal to the expiratory flow sensor. Auscultation of rhonchi or tactile fremitus over the trachea would confirm excess secretions as the problem. The correct answer is: assess the patient's need for suctioning

Which of the following patient instructions for using a dry powder inhaler (DPI) is correct? A. hold the device vertically after loading B. inhale rapidly for 1-2 seconds C. blow slowly into the device D. breath normally in/out of the device

The following general guidelines apply to effective use of a DPI: (1) never use a spacer or VHC with a DPI; (2) lips must be tightly sealed around the mouthpiece; (3) after loading, most DPIs must be held horizontally (to avoid loss of drug); (4) patient should inhale rapidly (> 60 L/min for 1-2 sec) and deeply; and (5) patient must exhale to room (not back into the device). The correct answer is: inhale rapidly for 1-2 seconds

Which of the following parameters would you set to establish the minute volume for a patient being ventilated in the volume control assist/control mode (VC, A/C)? exp. time,rate,vt,insp. flow

The minute volume (VE) during VC, A/C ventilation is determined by the respiratory rate or frequency (f) and tidal volume setting (VT), that is VE = f x VT The correct answer is: A

Which of the following is a key therapeutic objective in the management of a patient who has closed head trauma and is receiving ventilatory support? A. increase the minute ventilation B. increase intrathoracic pressure C. assure patient-ventilator synchrony D. decrease cerebral perfusion pressure

The overall goal of managing patients with closed head trauma is to prevent secondary injury by maintaining adequate cerebral perfusion pressure (CPP) and brain oxygenation. Because CPP = mean arterial pressure (MAP) - intracranial pressure (ICP), ventilatory care should aim to (1) maximize arterial oxygenation and (2) avoid actions that would either increase ICP or lower MAP. Goals therefore include maintaining an SaO2 of 100%; keeping the PaCO2 between 35-40 mm Hg (hypercapnia increases ICP); keeping the PIP ≤ 30 cm H2O (minimally affecting MAP); and assuring good patientventilator synchrony (helps prevent increases in intrathoracic pressure/ICP). Hyperventilation should only be considered if there is an acute deterioration in neurologic status that does not respond to standard brain trauma therapy, such as osmotic diuresis, CSF fluid drainage and sedation/neuromuscular blockage. The correct answer is: assure patient-ventilator synchrony

Which of the following is an indication for positive-end expiratory pressure? A. to provide graded levels of ventilatory support B. to decrease physiologic deadspace C. to decrease hypoxemia due to shunting D. to increase the efficiency of ventilation

The primary indication for PEEP is to decrease hypoxemia due to shunting in conditions like ARDS and IRDS. PEEP can also lower FIO2 needs in patients with refractory hypoxemia and help maintain or increase the FRC (e.g., in thoracic surgery). Last, extrinsic PEEP can be applied to decrease auto-PEEP in patients with airway obstruction receiving ventilatory support. The correct answer is: to decrease hypoxemia due to shunting

To help prevent infection after an aerosol drug treatment provided via small volume nebulizer (SVN) you would: A. shake out any residual solution then bag the SVN B. rinse the SVN with tap water then dry and bag it C. run the SVN at high flows until completely dry D. rinse the SVN with sterile water then dry and bag it

To minimize the likelihood of infection in patients receiving aerosol drug therapy via a small volume nebulizer (SVN), you should 1) use a different SVN for each patient, 2) change the SVN and tubing every 24 hours, and 3) perform thorough hand hygiene prior to each therapy session. It is also recommended that the nebulizer NOT be rinsed with tap water, but rather rinsed with sterile water and blown dry between uses. If rinsing with sterile water not feasible, rinse the device with filtered or tap water, then rinse with isopropyl alcohol and dry. The correct answer is: rinse the SVN with sterile water then dry and bag it

You are called to the ED to provide a bronchodilator treatment for a patient having a severe asthma attack. When quickly confirming the written order you find it contains some prohibited notations and thus could be read as either '.5 U albuterol by SVN' or 5 c.c albuterol by SVN" The prescribing physician is busy overseeing a code. You should: 272 A. wait until the physician is done with the code so you can clarify the improper notation and the correct order B. administer the treatment using the standard dosage (0.5 mL) and clarify the order as soon as possible thereafter C. cross out the prohibited notations, provide the correct abbreviations and initial and date the changes D. have the nurse review the order and correct the improper notation

This order contains at least two improper notations, i.e., c.c. (use mL), and lack of leading zeros before a decimal point (that may be 'lost' on the order line). Normally, if an order contains a prohibited notation, the respiratory therapist must confirm the intent of the order before proceeding. The exception is when order confirmation might delay essential or emergency patient treatment (as here). In these cases, if, in the judgment of the caregiver the order is clear and complete and the delay to obtain confirmation from the prescriber would place the patient at greater risk, then the order should be carried out and the confirmation obtained as soon as possible thereafter. The correct answer is: administer the treatment using the standard dosage (0.5 mL) and clarify the order as soon as possible thereafter

A 30 year-old male was found supine and unresponsive. In the ER it was confirmed he had aspirated while on his back. After the patient is transferred to ICU his physician orders postural drainage and percussion every 4 hours. What is the best position to place him in to drain the affected area? A. prone with a pillow under his hips B. prone with feet elevated 30 degrees C. supine with a pillow under his hips D. supine with feet elevated 30 degrees

This patient aspirated while lying flat on his back. Most commonly, this affects the superior segments of both lower lobes. The position which facilitates drainage from this lung region is a prone position with a pillow under the patient's hips. The correct answer is: prone with a pillow under his hips

A 52 year-old male is admitted to the hospital emergency room with a chief complaint of severe radiating chest pain and signs of central cyanosis. The attending physician orders STAT O2 therapy. Which of the following would you recommend? A. simple oxygen mask at 8 L/min B. air entrainment mask at 40% O2 C. partial rebreathing mask at 10 L/min D. nasal cannula at 5 L/min

When moderate to high FIO2s are needed quickly and for relatively short periods of time, a partial or nonrebreathing mask should be used. As the emergency O2 equipment of choice, disposable versions of these devices can provide FIO2s up to 0.60 to 0.70. The correct answer is: partial rebreathing mask at 10 L/m

A patient on mechanical ventilation is improving, and the physician wants to begin weaning her. Originally, she was on pressure controlled A/C with a rate of 12/min and a tidal volume of 600 mL. For weaning the doctor orders a switch to pressure support ventilation at 10 cm H2O. Initially, the patient seems comfortable on pressure support, but after 45 minutes she is moderately tachypneic and is starting to use her accessory muscles. Which of the following would you recommend to alleviate this problem? A. giving her 10 mg of Valium for anxiety B. putting her back on pressure control ventilation C. increasing the pressure support level to 15 cm H2O D. adding 5 cm H2O of PEEP

When a patient being weaned exhibits tachypnea and accessory muscle use, it is likely that the work of spontaneous breathing is too high. In these cases, you first action should be to increase the pressure support level. This will allow for a decrease in spontaneous work of breathing, and a more efficient breathing pattern (higher tidal volume and lower rate). The correct answer is: increasing the pressure support level to 15 cm H2O

An adult patient is intubated after being pulseless for several minutes. An exhaled CO2 detection device fails to change color despite confirmation of tracheal placement by auscultation and chest rise. A respiratory therapist should recommend: performing a direct laryngoscopy. replacing the CO2 detection device. obtaining a stat chest radiograph. performing pulse oximetry.

c) A. Direct visualization of the larynx with a laryngoscope will confirm the tube has passed through the cords. (h) B. It is unlikely that the CO2 detector is not functioning correctly. Since the patient has been pulseless for several minutes, the amount of CO2 exhaled will be significantly reduced as the pulmonary blood flow is minimal. (h) C. A chest radiograph will confirm placement of the endotracheal tube; however, an unacceptable amount of time would be required to complete this procedure in a pulseless patient. (h) D. Pulse oximetry will offer no guidance with this pulseless patient.

A male patient who is experiencing an acute myocardial infarction will most likely have which of the following clinical findings? jaw pain pedal edema nausea and vomiting fever

c) A. Jaw pain with nausea and vomiting are classic signs that a male patient is experiencing an acute myocardial infarction. (u) B. Pedal edema and fever are not indicators of an acute myocardial infarction. (u) C. Fever is not an indicator of an acute myocardial infarction. (u) D. Pedal edema is not an indicator of an acute myocardial infarction.

A 188-cm (6-ft, 2-in), 80-kg (176-lb) male patient who has undergone a right lower lobectomy is receiving VC, A/C ventilation with the following settings: FIO2 0.50 Mandatory rate 12 Total rate 14 VT 400 mL The following blood gas results are available: pH 7.32 PaCO2 48 torr PaO2 75 torr HCO3- 25 mEq/L BE -2 mEq/L A respiratory therapist should recommend: changing to SIMV. initiating PEEP of 10 cm H2O. maintaining current therapy. increasing the tidal volume.

u) A. Changing to SIMV and retaining the same settings will likely result in a reduction in ventilation. (h) B. PEEP of 10 cm H2O is not indicated because of the presence of a bronchial stump. (c) C. Although the patient has a mild respiratory acidosis, the values are acceptable for this patient. (h) D. Increasing the tidal volume is not indicated due to the presence of a bronchial stump.

An air-entrainment mask will deliver an FIO2 higher than intended if: the flow is set too high. nebulized water is being added through the air-entrainment ports. corrugated tubing was added between the air-entrainment adapter and mask.r the air-entrainment ports have been blocked.

u) A. Increasing the flow will entrain more room air while maintaining the same FIO2. (u) B. Adding nebulized water will have no effect on the FIO2. (u) C. Additinal tubing placed between the mask and air entrainment adapter will have no effect on the FIO2. (c) D. Blocked air entrainment ports prevents air from being added to the inspired gas flow and results in a higher FIO2.

A patient with COPD is receiving PC ventilation with flow triggering and has significant air trapping displayed on ventilator graphics. The patient's spontaneous breathing efforts are not always detected by the ventilator. Which of the following changes should a respiratory therapist recommend to improve patient-ventilator synchrony? Switch to pressure triggering. Switch to a square-wave flow pattern. Increase peak inspiratory flow. Increase the set PEEP.

u) A. Research has shown flow triggering to be more sensitive than pressure triggering in most cases. (u) B. A square-wave flow pattern cannot be used in the PC mode. The decelerating flow pattern generated during the PC mode has been found to improve synchrony when compared with a square-wave pattern. (u) C. The peak inspiratory flow cannot be independently adjusted in the PC mode. The flow is variable in this mode. (c) D. A patient who demonstrates patient-ventilator dyssynchrony associated with air trapping will often benefit from an increase in the set (extrinsic) PEEP level. The increase in applied PEEP can help reduce the difference between end alveolar pressure and end-inspiratory pressure.

In the last 6 months, a patient with bronchiectasis who uses postural drainage at home has had three exacerbations requiring hospitalization. Which of the following should a respiratory therapist recommend? insufflation/exsufflation device nebulized ipratropium bromide (Atrovent) inhaled corticosteroid HFCWO

u) A. Secretion clearance will not directly improve with an insufflation/exsufflation device. The device acts as a cough in clearing secretions, but it does not loosen the secretions. (u) B. Nebulized ipratropium bromide is indicated for reversible airways bronchoconstriction. It is not helpful in secretion clearance. (u) C. Inhaled corticosteroids are useful for reducing airways inflammation. This therapy will provide little benefit for clearing airways secretions. (c) D. HFCWO provides an effective method to loosen and mobilize airways secretions.

While reviewing a medical record, a respiratory therapist notes a patient has shortness of breath, pleuritic chest pain, low-grade fever, tachypnea, tachycardia, and a swollen and tender right leg. The patient has a 50 pack-year history of smoking and known coronary artery disease. These findings are MOST consistent with: pneumonia. pulmonary embolism. myocardial infarction. acute exacerbation of COPD.

u) A. Temperature irregularities such as fever or a lower than normal body temperature, productive cough, chest pain on inspiration, shortness of breath and diaphoresis are symptoms associated with pneumonia. (c) B. The signs and symptoms are consistent with pulmonary embolism. (u) C. A patient experiencing a myocardial infarction will have nausea and/or vomiting, diaphoresis as well as pain radiating to the arms and/or shoulders, neck and back and/or epigastric, but not chest wall tenderness. (u) D. Pleuritic chest pain and swelling and tenderness in one or both legs are not associated with an exacerbation of COPD.


Related study sets

Public Speaking final study guide

View Set

Chapter 4.2: Niches and Community Interactions

View Set

chapter 1 anatomy and physiology

View Set

Терапевтична стоматологія

View Set